FINAL

Pataasin ang iyong marka sa homework at exams ngayon gamit ang Quizwiz!

A client is teaching a client who has a new prescription for hydrochlorothiazide for management of hypertension. Which of the following instructions should the nurse include? "Take this medication before bedtime." "Monitor for leg cramps." "Avoid grapefruit juice.' "Reduce intake of potassium-rich foods."

"Monitor for leg cramps."

Q: A client asks the nurse what the difference is between osteoarthritis (OA) and rheumatoid arthritis (RA). Which response is correct? "OA and RA are very similar. OA affects the smaller joints and RA affects the larger, weight-bearing joints." "OA is more common in women. RA is more common in men." "OA affects joints on both sides of the body. RA is usually unilateral." "OA is a noninflammatory joint disease. RA is characterized by inflamed, swollen joints."

"OA is a noninflammatory joint disease. RA is characterized by inflamed, swollen joints."

Q: The nurse is beginning the assessment of a patient with an endocrine disorder. What should the nurse include in this assessment? Select all that apply. 1. Height and weight 2. Skin, hair, and nails 3. Deep tendon reflexes 4. Musculoskeletal system 5. Respiratory system 1,2,3,4,5 1,2,3,4 1,3,4,5 1,2,3,5

1,2,3,4

Q: "A client is admitted to the hospital with signs and symptoms of diabetes mellitus. Which findings is the nurse most likely to observe in this client? Select all that apply:" 1. Excessive thirst 2. Weight gain 3. Constipation 4. Excessive hunger 5. Urine retention 6. Frequent, high-volume urination

1,4,6

A nurse is caring for a client who recently started alteplase therapy. The nurse should monitor the client for which of the following adverse effects? 1- Headache 2 - tachycardia 3- hypotension 4- tinnitus

1- Headache

57. The nurse administers the wrong dose of a medication. Instead of giving furosemide 20mg orally, the nurse forgets to break the tablet in half and gives furosemide 40mg orally. The client experienced no harm as a result. What should the nurse do? SATA a. Continue to monitor the client's vital signs and urinary output. b. Notify the healthcare provider, supervisor, and client. c. Complete an incident report, outlining the events of the incident d. Document the time and amount of medication given. e. Nothing, because no harm came to the client and no further action is needed.

A,B,C Rationale: The medication given was double the amount prescribed, and it is important to document the amount given and continue to monitor the client. An incident report needs to be completed that outlines the events, provides the client status, and documents that all parties involved were notified of the events and the client's current status. The incident report is a process used by health care organizations to monitor events that occur. Documenting this information on the client's chart allows a legal team the option to subpoena the document. Doing nothing prevents a health care organization from analyzing the near misses or errors that occur and to facilitate identification of system processes that need to be changed.

Q: The nurse is caring for a client diagnosed with osteomyelitis. Which data noted in the client's record are supportive of this diagnosis? Select all that apply. A Pyrexia B Elevated potassium level C Elevated white blood cell count D Elevated erythrocyte sedimentation rate E Bone scan impression indicative of infection

A,C,D,E

In assessing the joints of a patient with rheumatoid arthritis, the nurse understands that the joints are damaged by (select all that apply). a. bony ankylosis following inflammation of the joints b. the deterioration of cartilage by proteolytic enzymes c. the development of Heberden's nodes in the joint capsule d.. increased cartilage and bony growth at the joint margins e. invasion of pannus into the joint causing a loss of cartilage

A,E

Q: Which condition is the leading cause of disability and pain in the elderly? A.Osteoarthritis (OA) B.Rheumatoid arthritis (RA) C.Systemic lupus erythematous (SLE) D.Scleroderma

A.Osteoarthritis (OA)

Q: A client is receiving the cell cycle-nonspecific alkylating agent thiotepa (Thioplex), 60 mg weekly for 4 weeks by bladder installation as part of chemotherapy regimen to treat bladder cancer. The client asks the nurse how the drug works. How does thiotepa exert its therapeutic effects? A.) It interferes with deoxyribonucleic acid (DNA) replication only. B.) It interferes with ribonucleic acid (RNA) transcription only. C.) It interferes with DNA replication and RNA transcription. D.) It destroys the cell membrane, causing lysis.

ANS C: It interferes with DNA replication and RNA transcription R: Thiotepa interferes with DNA replication and RNA transcription. It doesn't destroy the cell membrane.

Which of the following are possible complications following a Percutaneous Transluminal Coronary Angioplasty (PTCA)? Select all that apply: A) Hematoma formation B) Acute Renal Failure C) Hypothermia D) Cardiac Tamponade E) Bradycardia

ANS: A, B, D

A client with chronic renal failure (CRF) is admitted to the urology unit. Which diagnostic test results are consistent with CRF? Uric acid analysis 3.5 mg/dL and phenolsulfonphthalein (PSP) excretion 75% Blood urea nitrogen (BUN) 100 mg/dL and serum creatinine 6.5 mg/dL Increased pH with decreased hydrogen ions Increased serum levels of potassium, magnesium, and calcium

ANS: B - Blood urea nitrogen (BUN) 100 mg/dL and serum creatinine 6.5 mg/dL

Q: A health care provider orders tests to determine if a client has systemic lupus erythematosus (SLE). Which test result helps to confirm an SLE diagnosis? Increased total serum complement levels Negative antinuclear antibody test An above-normal anti-deoxyribonucleic acid (DNA) test Negative lupus erythematosus cell test

An above-normal anti-deoxyribonucleic acid (DNA) test

Q: The side effect of bone marrow depression may occur with which medication used to treat gout?

Ans: Allopurinol

A patient with AML is having aggressive chemotherapy to attempt to achieve remission. The patient is aware that hospitalization will be necessary for several weeks. What type of therapy will the nurse explain that the patient will receive? Induction therapy Supportive therapy Antimicrobial therapy Standard therapy

Answer: A - Induction therapy

A client is scheduled for a cholecystogram for later in the day. What is the nurse's understanding on the diagnostic use of this exam? a. It visualizes the gallbladder and bile duct b. It visualizes the liver and pancreas c. It shows the sizes of the abdominal organs and detects any masses d. It visualizes the biliary structures and pancreas via endoscopy

Answer: A - It visualizes the gallbladder and bile duct.

A client has been experiencing severe pain and hematuria and is hardly able to ambulate into the physician's office. The physician suspects kidney stones and orders diagnostic tests to confirm. What test would physician order? a. KUB b. Ultrasound c. CT d. MRI

Answer: A - KUB (X-ray of the kidneys, ureters and bladder)

A client develops acute renal failure (ARF) after receiving IV therapy with a nephrotoxic antibiotic. Because the client's 24-hour urine output totals 240 mL, the nurse suspects that the client is at risk for: a. cardiac arrhythmia b. paresthesia c. dehydration d. pruritus

Answer: A - cardiac arrhythmia

A client comes into the emergency department reporting an enlarged tongue. The tongue appears smooth and beefy red in color. The nurse also observes a 5-cm incision on the upper left quadrant of the abdomen. When questioned, the client states, "I had a partial gastrostomy 2 years ago." Based on this information, the nurse attributes these symptoms to which problem? Vitamin B12 deficiency Vitamin A deficiency Folic acid deficiency Vitamin C deficiency

Answer: A- Vitamin B-12 deficiency

A client is receiving radiation therapy for lesions in the abdomen from non-Hodgkin's lymphoma. Because of the effects of the radiation treatments, what will the nurse assess for? Hair loss Diarrheal stools Adventitious lung sounds Laryngeal edema

Answer: B - Diarrheal stools

A patient diagnosed AKI has a serum potassium level of 6.5 mEq/L. The nurse anticipates administering which of the following? a) Calcium supplements b) Kayexalate c) IV dextrose 50% d) Sorbitol

Answer: B - Kayexalate

A client presents with severe diarrhea and a history of chronic renal failure to the emergency department. Arterial blood gas results are as follows: pH 7.30, PaO2 97, PaCO2 37, HCO3 18. The nurse would expect which of the following sets of assessment findings? Headache, blood pressure 90/54, dry skin Blood pressure 188/120, nausea, vomiting Confusion, respiratory rate 8 breaths/min, dry skin Clammy skin, blood pressure 86/46, headache

Answer: D - clammy skin, blood pressure 86/46, headache

A client with advanced cirrhosis has a prothrombin time (PT) of 15 seconds, compared with a control time of 11 seconds. The nurse expects to administer: a. spironolactone (Aldactone) b. phytonadione (Mephyton) c. furosemide (Lasix) d. warfarin (Coumadin)

Answer: Phytonadione (Mephyton).

A confused client exhibits a blood pressure of 112/84, pulse rate of 116 beats per minute, and respiration's of 30 breaths per minute. The client's skin is cold and clammy. The nurse next A. Administers oxygen by nasal cannula at 2 liters per minute B. Re-assesses the vital signs C. Calls the Rapid Response Team D.Contacts the admitting physician

Answer: a. Administers oxygen by nasal cannula at 2 liters per minute

A nurse is preparing a client for cardiac catheterization. The nurse knows that which nursing intervention must be provided when the client returns to the room after the procedure? Withhold analgesics for at least 6 hours after the procedure. Assess the puncture site frequently for hematoma formation or bleeding. Inform the client that he or she may experience numbness or pain in the leg. Restrict fluids for 6 hours after the procedure.

Assess the puncture site frequently for hematoma formation or bleeding.

Q: Which of the following categories of oral antidiabetic agents exert their primary action by directly stimulating the pancreas to secrete insulin? A. Alpha glucosidase inhibitors B. Sulfonylureas C. Thiazolidinediones D. Biguanides

B. Sulfonylureas

Q: A client has been admitted to the postsurgical unit following a thyroidectomy. To promote comfort and safety, how should the nurse best position the client? A. Side-lying with one pillow under the head B. Head of the bed elevated 30 degrees and no pillows placed under the head C. Semi-Fowler with the head supported on two pillows D. Supine, with a small roll supporting the neck

C. Semi-Fowler with the head supported on two pillows

Q: A nurse is performing a physical examination on client suspected of having an endocrine disorder. Which assessment finding might be indicative of a problem with the thyroid gland? Sudden weight loss without dieting Dilated pupils Cold intolerance Diarrhea

Cold intolerance

Normal Lab Values of : Creatinine Lactate

Creatinine 0.5-1.5 Lactate <2

Allopurinol (Zyloprim) has been ordered for a patient receiving treatment for gout. The nurse caring for this patient knows to assess the patient for bone marrow suppression, which may be manifested by which of the following diagnostic findings? A) Hyperuricemia B) Increased erythrocyte sedimentation rate C) Elevated serum creatinine D) Decreased platelets

D) Decreased platelets

Pts should be instructed to consume alcohol in moderation. To consume at a moderate drinking pattern, a female pt should be instructed to consume no more than: A. two drinks per day B. six drinks per week C. three drinks per week D. one drink per day

D. one drink per day

A patient is admitted with suspected cardiomyopathy. What diagnostic test would be most helpful with the identification of this disorder? Cardiac catheterization Phonocardiogram Echocardiogram Serial enzyme studies

Echocardiogram

Q: When reviewing laboratory results for a patient with a possible diagnosis of hypoparathyroidism, the nurse knows that this condition is characterized by which of the following? Inadequate secretion of parathormone Increase in serum calcium Lowered blood phosphate Increase in the renal excretion of phosphate

Inadequate secretion of parathormone

The nurse is caring for a patient with severe left ventricular dysfunction who has been identified as being at risk for sudden cardiac death. What medical intervention can be performed that may extend the survival of the patient? Insertion of an implantable cardioverter defibrillator Insertion of an implantable pacemaker Administration of a calcium channel blocker Administration of a beta-blocker

Insertion of an implantable cardioverter defibrillator

36. Which medication is the most effective agent in the treatment of Parkinson disease?

Levodopa

Q: A client is undergoing diagnostic testing for osteomalacia. Which of the following laboratory results are most suggestive of this diagnosis? A- High chloride, calcium, and magnesium levels B - High parathyroid and calcitonin levels C - Low serum calcium and magnesium levels D - Low serum calcium and low phosphorus level

Low serum calcium and low phosphorus level

Q; A medical nurse is aware of the need to screen specific patients for their risk of hyperglycemic hyperosmolar syndrome (HHS). In what patient population does hyperosmolar nonketotic syndrome most often occur? Patients who are obese and who have no known history of diabetes Patients with type 1 diabetes and poor dietary control Adolescents with type 2 diabetes and sporadic use of antihyperglycemics Middle-aged or older people with either type 2 diabetes or no known history of diabetes

Middle-aged or older people with either type 2 diabetes or no known history of diabetes

As a client approaches death, respirations become noisy. This is the result of which type of physical event? Musculoskeletal change cardiac dysfunction central nervous system alterations gastrointestinal impairment

Musculoskeletal change

The nurse is teaching a client diagnosed with coronary artery disease about nitroglycerin. What is the cardiac premise behind administration of nitrates? Preload is reduced. More blood returns to the heart. It increases myocardial oxygen consumption. It functions as a vasoconstrictor.

Preload is reduced.

Q: A hospitalized client with terminal heart failure is nearing the end of life. The nurse observes which of the following breathing patterns?

Rationale: Cheyne-Stokes breathing is characterized by a regular cycle where the rate and depth of breathing increase, then decrease until apnea occurs. The duration of apnea varies but progresses in length. This breathing pattern is associated with heart failure, damage to the respiratory center in the brain, or both.

Q: A client is transferred to a rehabilitation center after being treated in the hospital for a stroke. Because the client has a history of Cushing's syndrome (hypercortisolism) and chronic obstructive pulmonary disease, the nurse formulates a nursing diagnosis of: Risk for imbalanced fluid volume related to excessive sodium loss. Risk for impaired skin integrity related to tissue catabolism secondary to cortisol hypersecretion. Ineffective health maintenance related to frequent hypoglycemic episodes secondary to Cushing's syndrome. Decreased cardiac output related to hypotension secondary to Cushing's syndrome.

Risk for impaired skin integrity related to tissue catabolism secondary to cortisol hypersecretion.

33. A client seeks medical attention for a new onset of fatigue and changes in coordination. Which additional assessment finding indicates to the nurse that the client is demonstrating signs of low oxygenation? Select all that apply. Cough Shortness of breath Drowsiness Impaired thought process Agitation

Shortness of breath, Impaired thought process, Agitation

A nurse is educating a client about monitoring blood pressure readings at home. Which of the following will the nurse be sure to emphasize? Sit with legs crossed when taking your blood pressure Sit quietly for 5 minutes prior to taking blood pressure Be sure the forearm is well supported above heart level while taking blood pressure Avoid smoking cigarettes for 8 hours prior to taking blood pressure

Sit quietly for 5 minutes prior to taking blood pressure

The nurse is preparing the procedure room for a client who will undergo an intravenous pyelogram. Which item(s) should the nurse include? Antihypertensive agents Dressings and tape Suction equipment Padded tongue blades

Suction equipment

The client asks the nurse why a stress test is needed. What statement best explains the rationale for the health care provider to order a cardiac stress test? The health care provider wants to identify if the heart failure is from coronary artery disease. The health care provider needs to evaluate everything. Heart failure is causing the client to be weak and tired. The stress test is the best diagnostic tool to monitor which stage of heart failure the client is experiencing.

The health care provider wants to identify if the heart failure is from coronary artery disease.

Q: The adrenal medulla secretes epinephrine during sympathetic stimulation. Which of the following statements describes why epinephrine would increase the sympathetic response? The same receptors that respond to norepinephrine also respond to epinephrine, increasing the action of both. Epinephrine has no effect on sympathetic targets. Epinephrine increases the action of pre- and postganglionic axons. Epinephrine binds to a separate receptor than norepinephrine, which increases the sympathetic response

The same receptors that respond to norepinephrine also respond to epinephrine, increasing the action of both.

Q: Determine whether the following statement is true or false. The sympathetic division causes the adrenal medulla to release epinephrine and norepinephrine.

True

Before a transesophageal echocardiogram, a nurse gives a client an oral topical anesthetic spray. When the client returns from the procedure, the nurse observes no active gag reflex. What nursing action is a priority? Insert an oral airway. Withhold food and fluids. Position the client on his side. Introduce a nasogastric (NG) tube.

Withhold food and fluids.

Q: A nurse is teaching a female client about preventing osteoporosis. Which teaching point is correct? a) The recommended daily allowance of calcium may be found in a wide variety of foods. b) Obtaining the recommended daily allowance of calcium requires taking a calcium supplement. c) To prevent fractures, the client should avoid strenuous exercise. d) Obtaining an X-ray of the bones every 3 years is recommended to detect bone loss.

a) The recommended daily allowance of calcium may be found in a wide variety of foods.

56. A graduate nurse is preparing to start a first nursing job. What action would be the best legal safeguard for the graduate nurse to take? a. Competent practice c. A valid license b. A legal contract d. Following management policies

a. Competent practice

The nurse is teaching a client about the functionality of heart muscle. What factor may decrease a client's myocardial contractility? acidosis alkalosis sympathetic activity administration of digoxin

acidosis

When should the nurse plan the rehabilitation of a patient who is having an ischemic stroke? a) After the patient has passed the acute phase of the stroke b) After the nurse has received the discharge orders c) The day the patient has the stroke d) The day before the patient is discharged

c) The day the patient has the stroke

Chronic obstructive pulmonary diseases (COPD) are diseases of the airways that produce obstruction to expiratory airflow. Which of the following problems relates to COPD? a. Inflammation of mucosal lining of airways b. Retained secretions c. Bronchial constriction related to spasm of the smooth muscle d. All of the above

d. All of the above

22. A client has just undergone bronchoscopy. Which nursing assessment is most important at this time? a. Voice quality b. Swallowing reflex c. Anxiety d. Level of consciousness (LOC)

d. Level of consciousness (LOC)

47. The nurse is performing an initial assessment on a client admitted to rule out Guillain-Barre syndrome. On which of the following areas will the nurse focus most heavily? a. Gastrointestinal b. Urinary c. Skin d. Respiratory

d. Respiratory Rationale: Because of its possible rapid progression and neuromuscular respiratory failure, Guillain-Barre syndrome is a medical emergency. After baseline values are identified, assessment of changes in muscle strength and respiratory function alerts the team to the physical and respiratory needs of the client. The other three choices may become problem areas later, but respiratory issues are always a priority.

Q: If your client's urine output is _______ and his specific gravity is ______, it means the pt is dehydrated.

low, high

A male client has been receiving a continuous infusion of weight-based heparin for more than 4 days. The client's PTT is at a level that requires an increase of heparin by 100 units per hour. The client has the laboratory findings shown above. What is the most important action for the nurse to take? Continue with the present infusion rate of heparin. Consult with the physician about discontinuing heparin. Begin treatment with the prescribed warfarin (Coumadin). Increase the heparin infusion by 100 units per hour. A

nswer: B - Consult with the physician about discontinuing heparin.

Q: A nurse is caring for a client with suspected hyperparathyroidism. Which condition may contribute to hyperparathyroidism? Renal failure Thyroidectomy Decreased serum calcium level Steroid use

renal failure

Q: The home care nurse is planning the order of clients for the day. Which client should the nurse prioritize as needing to be seen first​? A newly diagnosed diabetic client who is administering morning insulin independently for the first time A client with daily dressing​ change, normally done at 0800 per client preference A client being seen poststroke for rehabilitation and education about post stroke care A client requiring indwelling catheter change due to leakage​

A newly diagnosed diabetic client who is administering morning insulin independently for the first time

The nurse is performing a physical assessment on a patient suspected of having HF. The presence of what sound would signal the possibility of impending HF? A) An S3 heart sound B) Pleural friction rub C) Faint breath sounds D) A heart murmur

A) An S3 heart sound

Q: A patient with thyroid cancer has undergone surgery and a significant amount of parathyroid tissue has been removed. The nurse caring for the patient should prioritize what question when addressing potential complications? A) Do you feel any muscle twitches or spasms? B) ìDo you feel flushed or sweaty?î C) ìAre you experiencing any dizziness or lightheadedness?î D) ìAre you having any pain that seems to be radiating from your bones?î

A) Do you feel any muscle twitches or spasms?

13. What disease process(es) contributes to chronic heart failure? Select all that apply. A) Tachydysrhythmias B) Valvular disease C) Pancreatic disease D) Renal failure E) Pulmonary insufficiency

A) Tachydysrhythmias B) Valvular disease D) Renal failure

Q: A patient has a diagnosis of rheumatoid arthritis and the primary care provider has now prescribed cyclophosphamide (Cytoxan). The nurse's subsequent assessments should address what potential adverse effect? A) Infection B) Acute confusion C) Sedation D) Malignant hyperthermia

A) Infection

Q: A patient presents at the walk-in clinic complaining of diarrhea and vomiting. The patient has a documented history of adrenal insufficiency. Considering the patient's history and current symptoms, the nurse should anticipate that the patient will be instructed to do which of the following? A)Increase his intake of sodium until the GI symptoms improve. B) Increase his intake of potassium until the GI symptoms improve. C) Increase his intake of glucose until the GI symptoms improve. D) Increase his intake of calcium until the GI symptoms improve.

A)Increase his intake of sodium until the GI symptoms improve.

A nurse is educating a client with coronary artery disease about nitroglycerin administration. The nurse tells the client that nitroglycerin has what actions? Select all that apply. A - Reduces myocardial oxygen consumption B - Decreases the urge to use tobacco C - Dilates blood vessels D - Decreases ischemia E - Relieves pain

A, C, D, E Reduces myocardial oxygen consumption, dilates blood vessels, decreases ischemia, relieves pain

Q: A client with a terminal illness is diagnosed with sarcopenia (loss of muscle and strength). Which mechanism(s) of the anorexia/cachexia syndrome does the nurse identify that contributed to the development of this condition? Select all that apply. a. Reduced rate of muscle protein synthesis b. Decreased immune response c. Loss of appetite d. Intolerance to treatments e. Reduced voluntary motor activity

A,E

A client has received several treatments of bleomycin. It is now important for the nurse to assess? a) Urine output b) Lung sounds c) Skin integrity d) Hand grasp

ANS B: Lung sounds R: Bleomycin has cumulative toxic effects on lung function. Thus, it will be important to assess lung sounds.

Because of difficulties with hemodialysis, peritoneal dialysis is initiated to treat a client's uremia. Which finding during this procedure signals a significant problem? Blood glucose level of 200 mg/dl White blood cell (WBC) count of 20,000/mm3 Potassium level of 3.5 mEq/L Hematocrit (HCT) of 35%

ANS: B - White blood cell (WBC) count of 20,000/mm3

20. The nurse notes that a client has several normal breaths that are followed by apnea that varies in length. The nurse knows that this breathing pattern is also known as? a. Biot respirations b. Obstructive c. Hyperpnea d. Cheyne-Stokes

a. Biot respirations Rationale: Biot respirations are characterized by periods of normal breathing followed by varying periods of apnea. This breathing pattern is also called ataxic breathing and is associated with a drug overdose or brain injury at the level of the medulla.

48. A client with dilated cardiomyopathy is having frequent episodes of ventricular fibrillation. What medical treatment does the nurse anticipate the client will have to terminate the episode of ventricular fibrillation? internal cardioverter defibrillator (ICD) insertion pacemaker insertion radiofrequency ablation electrophysiological study

internal cardioverter defibrillator (ICD) insertion

An elderly man presents with slurred speech, confusion, and unilateral facial asymmetry. When asked to squeeze your hands, the strength in his left hand is markedly less than the strength in his right hand. The patient's wife tells you that her husband has type 2 diabetes and hypertension. On the basis of your clinical findings, you should: start an IV and administer crystalloid fluid boluses if his systolic blood pressure is less than 110 mm Hg. rule out hypoglycemia by assessing his blood sugar, but suspect a right-sided ischemic stroke. suspect that he is experiencing a hemorrhagic stroke, begin immediate transport, and start an IV en route. consider him a candidate for fibrinolytic therapy if his symptoms began less than 12 hours ago.

rule out hypoglycemia by assessing his blood sugar, but suspect a right-sided ischemic stroke.

Q: A nurse is teaching a client who has RA about taking methotrexate. Which of the following information should the nurse include?

A: Drink 2 to 3 L of water per day. R: Methotrexate can cause renal toxicity. The client should drink 2 to 3 L of water per day to promote excretion of the medication.

A bowel resection is scheduled for a client with the diagnosis of colon cancer with metastasis to the liver and bone. Which statement by the nurse best explains the purpose of the surgery? "Removing the tumor is a primary treatment for colon cancer." "This surgery will prevent further tumor growth." "Once the tumor is removed, cell pathology can be determined." "Tumor removal will promote comfort."

ANS: "Tumor removal will promote comfort." R: Palliative surgeries, such as bowel resection, may be performed to promote comfort by relieving pain and pressure on organs within the abdominal cavity. Primary treatment refers to surgery that is likely to provide a cure, which is not likely in metastatic disease. With metastasis, primary tumor removal does not prevent further tumor growth in distant sites.

An 80-year-old man in a long-term care facility has a chronic leg ulcer and states that the area has become increasingly painful in recent days. The nurse notes that the site is now swollen and warm to the touch. The client should undergo diagnostic testing for what health problem? A. Osteomyelitis B. Osteoporosis C. Osteomalacia D. Septic arthritis

ANS: A Rationale: When osteomyelitis develops from the spread of an adjacent infection, no signs of septicemia are present, but the area becomes swollen, warm, painful, and tender to touch. Osteoporosis is noninfectious. Osteomalacia is a metabolic bone disease characterized by inadequate mineralization of bone. Septic arthritis occurs when joints become infected through spread of infection from other parts of the body (hematogenous spread) or directly through trauma or surgical instrumentation.

The nurse is planning an education program for women of childbearing years. The nurse recognizes that primary prevention of osteoporosis includes:

ANS: Ensuring adequate calcium and vitamin D intake

A patient with systemic lupus erythematosus (SLE) is preparing for discharge. The nurse knows that the patient has understood health education when the patient makes what statement? A) Ill make sure I get enough exposure to sunlight to keep up my vitamin D levels. B) Ill try to be as physically active as possible between flare-ups. C) Ill make sure to monitor my body temperature on a regular basis. D) Ill stop taking my steroids when I get relief from my symptoms.

Ans: C: I'll make sure to monitor my body temperature on a regular basis. Professor R: Fever can signal an exacerbation in a patient with SLE and should be reported to the health care provider. Sunlight and other sources of ultraviolet light may precipitate severe skin reactions and exacerbate the disease. Fatigue can cause a flare-up of SLE. Clients should be encouraged to pace activities and plan rest periods. Corticosteroids must be gradually tapered because they can suppress the function of the adrenal gland. As well, these drugs should not be independently adjusted by the client.

Q: A client is recovering from an attack of gout. What will the nurse include in the client's teaching?

Ans: Weight loss will reduce uric acid levels and reduce stress on joints.

A client with acute pancreatitis has jaundice with diminished bowel sounds and a tender distended abdomen. Additionally, lab results indicate hypovolemia. What will the physician order to treat the large amount of protein-rich fluid that has been released into the client's tissues and peritoneal cavity? Select all that apply. a. albumin b. sodium c. dextrose solution d. diuretics

Answer: A (albumin), D (diuretics)

A client in the emergency department reports squeezing substernal pain that radiates to the left shoulder and jaw. The client also complains of nausea, diaphoresis, and shortness of breath. What is the nurse's priority action? Administer oxygen, attach a cardiac monitor, take vital signs, and administer sublingual nitroglycerin. Alert the cardiac catheterization team, administer oxygen, attach a cardiac monitor, and notify the health care provider. Gain I.V. access, give sublingual nitroglycerin, and alert the cardiac catheterization team. Complete the client's registration information, perform an electrocardiogram, gain I.V. access, and take vital signs.

Answer: A - Administer oxygen, attach a cardiac monitor, take vital signs, and administer sublingual nitroglycerin.

A client with AML has pale mucous membranes and bruises on the legs. What is the primary nursing intervention? Assess the client's hemoglobin and platelets. Assess the client's pulse and blood pressure. Check the client's history. Assess the client's skin.

Answer: A - Assess the client's hemoglobin and platelets.

______ is the end product of muscle metabolism. It is a better indicator of renal function than BUN because it does not vary with protein intake and metabolic state.

Answer: Creatinine

The physical therapist notifies the nurse that a client with coronary artery disease (CAD) experienced a significant increase in heart rate during physical therapy. The nurse recognizes that an increase in heart rate in a client with CAD may result in which outcome? A. Development of an atrial-septal defect B. Myocardial ischemia C. Formation of a pulmonary embolism D. Release of potassium ions from cardiac cells

B. Myocardial ischemia

Q: A nurse is caring for a client with a warm and painful toe from gout. What medication will the nurse administer? A.furosemide B.Colchicine C.calcium gluconate D.Aspirin

B.Colchicine

Q: A client has returned to the floor after having a thyroidectomy for thyroid cancer. What laboratory finding may be an early indication of parathyroid gland injury or removal? A. Hyponatremia B. Hypophosphatemia C. Hypocalcemia D. Hypokalemia

C. Hypocalcemia

Q: In teaching a patient with SLE about the disorder, the nurse knows that the pathophysiology of SLE includes: A. Circulating immune complexes formed from IgG autoantibodies reacting with IgG B. An autoimmune T-cell reaction that results in destruction of the deep dermal skin layer C. Immunologic dysfunction leading to chronic inflammation in the cartilage and muscles D. The production of a variety of autoantibodies directed against components of the cell nucleus

D. The production of a variety of autoantibodies directed against components of the cell nucleus

A client has been prescribed a beta blocker. The nurse knows that beta blockers can have which effect on the heart? Increase the heart rate Decrease the heart rate Prevent normal sinus rhythm Constrict the heart

Decrease the heart rate

Which of the following are usually the first choice in the treatment of rheumatoid arthritis (RA)? Nonsteroidal anti-inflammatory drugs (NSAIDs) Disease-modifying antirheumatic drugs (DMARDs) Tumor necrosis factor (TNF) blockers Glucocorticoids

Disease-modifying antirheumatic drugs (DMARDs)

Q: A diabetes nurse educator is teaching a group of patients with type 1 diabetes about sick day rules. What guideline applies to periods of illness in a diabetic patient? Do not eliminate insulin when nauseated and vomiting. Report elevated glucose levels greater than 150 mg/dL. Eat three substantial meals a day, if possible. Reduce food intake and insulin doses in times of illness.

Do not eliminate insulin when nauseated and vomiting.

Q: A client with rheumatoid arthritis is prescribed a tumor necrosis factor (TNF)-alpha inhibitor. Which of the following might be prescribed? Diclofenac Indomethacin Etanercept Celecoxib

Etanercept

The nurse is preparing to administer hydralazine and isosorbide dinitrate. When obtaining vital signs, the nurse notes that the blood pressure is 90/60. What is the priority action by the nurse? Hold the medication and call the health care provider. Administer the medication and check the blood pressure in 30 minutes. Administer the hydralazine and hold the dinitrate. Administer a saline bolus of 250 mL and then administer the medication.

Hold the medication and call the health care provider.

A nurse is performing a cardiac assessment on an elderly client. Which finding warrants further investigation? Fourth heart sound (S4) Orthostatic hypotension Increased PR interval Irregularly irregular heart rate

Irregularly irregular heart rate

46. The nurse is taking health history from a client admitted to rule out Guillain-Barre syndrome. An important question to ask related to the diagnosis is which of the following? a. "Have you experienced any viral infections in the last month?" b. "Have you experienced any ptosis in the last few weeks?" c. "Have you had difficulty with urination in the last 6 weeks?" d. "Have you developed any new allergies in the last year?"

a. "Have you experienced any viral infections in the last month?" Rationale: An antecedent event (most often a viral infection) precipitates clinical presentation. The antecedent event usually occurs about 2 weeks before the symptoms begin. Ptosis is a common symptom associated with myasthenia gravis. Urination and development of allergies are not associated with Guillain-Barre.

31. A patient on the medical unit has told the nurse that he is experiencing significant dyspnea, despite that he has not recently performed any physical activity. What assessment question should the nurse ask the patient while preparing to perform a physical assessment? a. "On a scale from 1 to 10, how bad would you rate your shortness of breath?" b. "When was the last time you ate or drank anything?" c. "Are you feeling any nausea along with your shortness of breath?" d. "Do you think that some medication might help you catch your breath?"

a. "On a scale from 1 to 10, how bad would you rate your shortness of breath?"

21. A nurse has just completed teaching with a patient who has been prescribed a meter-dosed inhaler for the first time. Which of the following statements would the nurse use to initiate further teaching and follow-up care? a. "I will make sure to take a slow, deep breath as I push on my inhaler." b. "After I breathe in, I will hold my breath for 10 seconds." c. "I do not need to rinse my mouth with this type of inhaler." d. "If I use the spacer, I know I am only supposed to push on the inhaler once."

c. "I do not need to rinse my mouth with this type of inhaler."

Q: A nurse is providing dietary instructions to a client with diabetes. What is most important for the nurse to include in teaching for prevention of hypoglycemia? a. Increase protein intake in the morning. b. Reduce carbohydrate intake when drinking alcohol. c. Drink orange juice if lightheadedness occurs. d. Avoid delaying or skipping meals.

d. Avoid delaying or skipping meals.

27. A nurse is preparing a client with a pleural effusion for a thoracentesis. The nurse should: a. raise the head of the bed to a high Fowler's position. b. Raise the arm on the side of the client's body on which the physician will perform the thoracentesis. c. place the client supine in the bed, which is flat. d. assist the client to a sitting position on the edge of the bed, leaning over the bedside table.

d. assist the client to a sitting position on the edge of the bed, leaning over the bedside table.

A client returns for a follow-up visit to the cardiologist 4 days after a trip to the ED for sudden shortness of breath and abdominal pain. The nurse realizes the client had a myocardial infarction because the results from the blood work drawn in the hospital shows: elevated troponin levels. decreased LDH levels. decreased myoglobin levels. increased C-reactive protein levels.

elevated troponin levels.

Q: Acarbose (Precose), an alpha-glucosidase inhibitor, is prescribed for a female client with type 2 diabetes mellitus. During discharge planning, nurse Pauleen would be aware of the client's need for additional teaching when the client states: A- "If I have hypoglycemia, I should eat some sugar, not dextrose." B- "The drug makes my pancreas release more insulin." C- "I should never take insulin while I'm taking this drug." D- "It's best if I take the drug with the first bite of a meal."

A- "If I have hypoglycemia, I should eat some sugar, not dextrose."

Q: When reviewing laboratory results for a patient with a possible diagnosis of hypoparathyroidism, the nurse knows that this condition is characterized by which of the following? A- Inadequate secretion of parathormone B- Increase in serum calcium C- Lowered blood phosphate D- Increase in the renal excretion of phosphate

A- Inadequate secretion of parathormone

Choose the statements that correctly match the hypertensive medication with its side effect. Select all that apply. A-With thiazide diuretics, monitor serum potassium concentration. B- Direct vasodilators may cause headache and tachycardia. C - Cough is a common side effect of adrenergic inhibitors. D - Beta-blockers may cause sedation. E - With ACE inhibitors, assess for bradycardia.

A-With thiazide diuretics, monitor serum potassium concentration. B- Direct vasodilators may cause headache and tachycardia.

Q: Which statement about fluid replacement is accurate for a client with hyperosmolar hyperglycemic nonketotic syndrome? A. Administer 2 to 3 L of IV fluid rapidly. B. Administer 10 L of IV fluid over the first 24 hours. C. Administer a dextrose solution containing normal saline solution. D. Administer IV fluid slowly to prevent circulatory overload and collapse.

A. Administer 2 to 3 L of IV fluid rapidly.

Q: A nurse is caring for a client newly diagnosed with type 1 diabetes. The nurse is educating the client about self-administration of insulin in the home setting. The nurse should teach the client to do what action? A. Avoid using the same injection site more than once in 2 to 3 weeks. B. Avoid mixing more than one type of insulin in a syringe. C. Cleanse the injection site thoroughly with alcohol prior to injecting. D. Inject at a 45-degree angle.

A. Avoid using the same injection site more than once in 2 to 3 weeks.

Q: A client with type 1 diabetes has been on a regimen of multiple daily injection therapy. He's being converted to continuous subcutaneous insulin therapy. While teaching the client about continuous subcutaneous insulin therapy, the nurse should tell him that the regimen includes the use of: A. rapid-acting insulin only. B. short- and intermediate-acting insulins. C. intermediate- and long-acting insulins. D. short- and long-acting insulins.

A. rapid-acting insulin only.

3. The nurse is caring for a patient with a diagnosis of hyponatremia. What nursing intervention is appropriate to include in the plan of care for this patient? (Select all that apply.) -Restricting tap water intake -Assessing for symptoms of nausea and malaise -Encouraging the use of salt substitute instead of salt - Monitoring neurologic status -. Encouraging the intake of low-sodium liquids A- 1, 2, 4 B - 1, 3, 5 C - 3, 4, 5 D-1, 2, 3, 4

A: 1- Restricting tap water intake 2- . Assessing for symptoms of nausea and malaise 4- . Monitoring neurologic status

Q: Which of the following are usually the first choice in the treatment of rheumatoid arthritis (RA)? Nonsteroidal anti-inflammatory drugs (NSAIDs) Disease-modifying antirheumatic drugs (DMARDs) Tumor necrosis factor (TNF) blockers Glucocorticoids

ANS B: Disease-modifying antirheumatic drugs (DMARDs)

Q: Which patient assessment would indicate to the nurse that salicylate toxicity is occurring? a. Gastrointestinal (GI) bleeding b. Increased bleeding times c. Tinnitus d. Occasional nausea

ANS C: Tinnitus QUIZLET RATIONALE: Symptoms of salicylism include ringing in the ears (tinnitus), impaired hearing, dimming of vision, sweating, fever, lethargy, dizziness, mental confusion, nausea, and vomiting. Although Salicylates may cause GI bleeding over time, it is not a symptom associated with toxicity

Q: Allopurinol (Zyloprim) has been ordered for a patient receiving treatment for gout. The nurse caring for this patient knows to assess the patient for bone marrow suppression, which may be manifested by which of the following diagnostic findings? A) Hyperuricemia B) Increased erythrocyte sedimentation rate C) Elevated serum creatinine D) Decreased platelets

ANS D: Decreased platelets (Feedback: Thrombocytopenia occurs in bone marrow suppression. Hyperuricemia occurs in gout, but is not caused by bone marrow suppression. Increased erythrocyte sedimentation rate may occur from inflammation associated with gout, but is not related to bone marrow suppression. An elevated serum creatinine level may indicate renal damage, but this is not associated with the use of allopurinol.)

Q: The result of which diagnostic study is decreased in the client diagnosed with rheumatoid arthritis?

ANS: Red blood cell count

A client with liver and renal failure has severe ascites. On initial shift rounds, his primary nurse finds his indwelling urinary catheter collection bag too full to store more urine. The nurse empties more than 2,000 ml from the collection bag. One hour later, she finds the collection bag full again. The nurse notifies the physician, who suspects that a bladder rupture is allowing the drainage of peritoneal fluid. The physician orders a urinalysis to be obtained immediately. The presence of which substance is considered abnormal? a. Albumin b. Creatinine c. Urobilinogen d. Chloride

Answer: A - Albumin

A 37-year-old male patient presents at the emergency department (ED) complaining of nausea and vomiting and severe abdominal pain. The patient's abdomen is rigid, and there is bruising to the patients flank. The patient's wife states that he was on a drinking binge for the past 2 days. The ED nurse should assist in assessing the patient for what health problem? a. Severe pancreatitis with possible peritonitis b. Acute cholecystitis c. Chronic pancreatitis d. Acute appendicitis with possible perforation

Answer: A - Severe pancreatitis with possible peritonitis

A client presents to the emergency department complaining of a dull, constant ache along the right costovertebral angle along with nausea and vomiting. The most likely cause of the client's symptoms is: a. renal calculi b. an overdistended bladder c. interstitial cystitis d. acute prostatitis

Answer: A - renal calculi

What pathophysiological concept related to sickle cell disease predisposes a client with sickle cell disease to pneumonia? a.Damage to the spleen increases the risk for infection. b. Damage to the lymphatic system increases the risk for infection. c. Sequestration of sickled cells lead to infection in the area of sequestration. d. Sequestration of sickled cells lead to infection in the area distal to the sequestration.

Answer: A Damage to the spleen increases the risk for infection

The nursing instructor is teaching nursing students about myocardial contractility and ejection fractions. What diagnostic tests can determine client ejection fractions? Select all that apply. A - Echocardiogram B - Cardiac catheterization C - Magnetic resonance imaging D - Positron emission tomography scan E - Troponin levels

Answer: A, B, C - Echocardiogram, Cardiac Catheterization, MRI

The nurse is caring for a patient with acute myeloid leukemia (AML) with high uric acid levels. What medication does the nurse anticipate administering that will prevent crystallization of uric acid and stone formation? Allopurinol Filgrastim Hydroxyurea Asparaginase

Answer: A- Allopurinol (Zylomprim)

After undergoing a liver biopsy, a client should be placed in which position? Right lateral decubitus position Semi-Fowler's position Supine position Prone position

Answer: A- Right lateral decubitus position

The nurse is caring for a patient following extensive abdominal surgery. The patient develops an infection that is treated with IV gentamicin. After 4 days of treatment, the patient develops oliguria, and laboratory results indicate azotemia. The patient is diagnosed with acute tubular necrosis and transferred to the ICU. The patient is hemodynamically stable. Which of the following dialysis methods would be most appropriate for the patient? a. Peritoneal dialysis b. Hemodialysis c. Continuous arteriovenous hemofiltration (CAVH) d. Continuous venovenous hemofiltration (CVVH)

Answer: B - Hemodialysis

What type of cancer is the most common type of secondary malignancy in patients with Hodgkin's disease? Breast Lung Colon Bone

Answer: B - Lung

What is a major concern for the nurse when caring for a patient with chronic pancreatitis? a. Mental status changes b. Weight loss c. Nausea d. Pain

Answer: B - Weight loss

A patient was admitted to the hospital with the following lab values: hemoglobin 5 g/dL, abnormally shaped erythrocytes, leukocyte count 2000/mm3 with hypersegmented neutrophils and a platelet count of 48,000/mm3. The platelets appear abnormally large. A bone marrow biopsy was competed and revealed hyperplasia. Based on this information, the nurse determines that patient most likely has which of the following diagnoses? Hemolytic anemia Folic acid deficiency Thalassemia Sickle cell anemia

Answer: B - folic acid deficiency

All of the following are part of the 1-Hour Septic Bundle except: A - Administer broad-spectrum antibiotics B - Obtain blood cultures after administration of antibiotics C - Give vasopressors if MAP <= 65 mm Hg D - Measure lactate level

Answer: B - obtain blood cultures after administration of antibiotics

A female client with chronic renal failure (CRF) is receiving a hemodialysis treatment. After hemodialysis, the nurse knows that the client is most likely to experience: hematuria weight loss increased urine output increased blood pressure

Answer: B - weight loss

A nursing student has learned about many collaborative interventions to achieve pain relief for clients with acute pancreatitis. Which of the following are appropriate? Choose all that apply. a. Allow the client to have sips of clear liquids. b. Withhold oral feedings to limit the release of secretin. c. Encourage bed rest to decrease the client's metabolic rate. d. Teach the client about the correlation between alcohol intake and pain.

Answer: B, C, D

A community health nurse is caring for a patient whose multiple health problems include chronic pancreatitis. During the most recent visit, the nurse notes that the patient is experiencing severe abdominal pain and has vomited 3 times in the past several hours. What is the nurses most appropriate action? A) Administer a PRN dose of pancreatic enzymes as ordered. B) Teach the patient about the importance of abstaining from alcohol. C) Arrange for the patient to be transported to the hospital. D) Insert an NG tube, if available, and stay with the patient.

Answer: C - Arrange for the patient to be transported to the hospital.

A client undergoes renal angiography. The nurse prepares the client for the test and provides post-procedure care. Which intervention should the nurse provide to the client after renal angiography? a. Encourage the client to void b. Monitor the client for signs and symptoms of pyelonephritis c. Palpate the pulses in the legs and feet d. Assess for signs of electrolyte and water imbalance

Answer: C - Palpate the pulses in the legs and feet

The nurse is educating a patient about preparation for an IV urography. What should the nurse be sure to include in the preparation instructions? The patient will have enemas until the urine is clear The patient is restricted from eating or drinking from midnight until after the test The patient may have liquids before the test A liquid restriction for 8 to 10 hours before the test is required

Answer: C - The patient may have liquids before the test

A client with chronic renal failure (CRF) has developed faulty red blood cell (RBC) production. The nurse should monitor this client for: nausea and vomiting dyspnea and cyanosis fatigue and weakness thrush and circumoral pallor

Answer: C - fatigue and weakness

A nurse working in the medical intensive care unit has a client admitted with mitral stenosis. The nurse is precepting a graduate nurse and explains the pathophysiology of the condition. What statement made by the graduate nurse will reflect an appropriate understanding of the disease process? Select all that apply. A - "Mitral stenosis affects the coronary blood flow and can lead to angina." B - "Mitral stenosis is caused by an obstruction between the right atrium and ventricle." C - "There is a narrowing between the left atrium and left ventricle." D - "It is caused by a tear that leads to the lungs becoming congested." E - "Increased blood flow in the left atrium causes left atrial hypertrophy."

Answer: C, E

A client with sickle cell disease informs the nurse that he is having chest pain. The nurse hears the client coughing, wheezing, and breathing rapidly. What does the nurse suspect is occurring with this client? Vaso-occlusive crisis Pneumocystis pneumonia Acute chest syndrome Acute muscular strain

Answer: C- Acute chest syndrome

A client with Hodgkin disease had a bone marrow biopsy yesterday and reports aching at the biopsy site, rated a 5 (on a 1-10 scale). After assessing the biopsy site, which nursing intervention is most appropriate? Notify the physician Administer aspirin (ASA) 325 mg po, as ordered Administer acetaminophen 500 mg po, as ordered Reposition the client to a high Fowler position and continue to monitor the pain

Answer: C- Administer acetaminophen 500mg po, as ordered

A nurse is caring for a client with a history of GI bleeding, sickle cell anemia, and a platelet count of 22,000/μl. The client, who is dehydrated and receiving dextrose 5% in half-normal saline solution at 150 ml/hour, complains of severe bone pain and is scheduled to receive a dose of morphine sulfate. For which administration route should the nurse question an order? Oral Subcutaneous (subQ) I.M. I.V.

Answer: C- I.M.

What intervention does the nurse anticipate providing for the patient with ascites that will help correct the decrease in effective arterial blood volume that leads to sodium retention? a. Diuretic therapy b. Therapeutic paracentesis c. Platelet infusions d. Albumin infusion

Answer: D - Albumin infusion

A nurse is caring for a client with cholelithiasis. Which sign indicates obstructive jaundice? a. Elevated urobilinogen in the urine b. Straw-colored urine c. Reduced hematocrit d. Clay-colored stools

Answer: D - Clay-colored stools

After being in remission from Hodgkin's disease for 18 months, a client develops a fever of unknown origin. A healthcare provider orders a liver biopsy to rule out advancing Hodgkin's disease and infection. Twenty-four hours after the biopsy, the client has a fever, complains of severe abdominal pain, and seems increasingly confused. What should the nurse suspect? Perforation of the colon caused by the liver biopsy Normal postprocedural pain, with a change in the level of consciousness resulting from the pre-existing fever An allergic reaction to the contrast media used during the liver biopsy Peritonitis from bleeding in the liver caused by the liver biopsy

Answer: D - Peritonitis from bleeding in the liver caused by the liver biopsy

A client with chronic renal failure has a serum potassium level of 6.8 mEq/L. What should the nurse assess first? Blood pressure Respirations Temperature Pulse

Answer: D - Pulse

A nurse is assessing an elderly patient with gallstones. The nurse is aware that the patient may not exhibit typical symptoms, and that particular symptoms that may be exhibited in the elderly patient may include what? a. Fever and pain b. Chills and jaundice c. Nausea and vomiting d. Signs and symptoms of septic shock

Answer: D - Signs and symptoms of septic shock

What is used to decrease potassium level seen in acute renal failure? a. IV dextrose 50% b. Sodium polystyrene sulfonate (Kayexalate) c. Calcium supplements d. Sorbitol

Answer: D - Sodium polystyrene sulfonate

A nurse is assessing a patient with chronic kidney disease. The nurse finds that the patient has pallor, and the blood reports are suggestive of anemia. What could be the most likely cause of anemia? Excess blood loss Decreased intake of iron Decreased thyroid gland activity Decreased erythropoietin production

Answer: D - decreased erythropoietin production

A client with chronic renal failure has a serum potassium level of 6.8 mEq/L. What should the nurse assess first? Blood pressure Respirations Temperature Pulse

Answer: D - pulse

A client is brought to the ED reporting fatigue, large amounts of bruising on the extremities, and abdominal pain localized in the left upper quadrant. A health history reveals the client has been treated for a sore throat three times in the past 2 months. Laboratory tests indicate severe anemia, significant neutropenia, and thrombocytopenia. Based on the symptoms, what could be the client's diagnosis? Sickle cell anemia Iron deficiency anemia Hemolytic anemia Aplastic anemia

Answer: D- Aplastic anemia

A client lives with a diagnosis of sickle cell disease and receives frequent blood transfusions. The nurse should recognize the client's consequent risk of what complication of treatment? Hypovolemia Vitamin B12 deficiency Thrombocytopenia Iron overload

Answer: D- Iron overload

A nurse is participating in the emergency care of a patient who has just developed variceal bleeding. What intervention should the nurse anticipate? A. Infusion of intravenous heparin B. IV administration of albumin C. STAT administration of vitamin K by the intramuscular route D. IV administration of octreotide (Sandostatin)

Answer: D. IV administration of octreotide (Sandostatin)

A nurse is caring for a client with a history of GI bleeding, sickle cell anemia, and a platelet count of 22,000/μl. The client, who is dehydrated and receiving dextrose 5% in half-normal saline solution at 150 ml/hour, complains of severe bone pain and is scheduled to receive a dose of morphine sulfate. For which administration route should the nurse question an order? a. Oral b. Subcutaneous (subQ) c. I.M. d. I.V.

Answer: I.M.

A physician orders lactulose (Cephulac), 30 ml three times daily, when a client with cirrhosis develops an increased serum ammonia level. To evaluate the effectiveness of lactulose, the nurse should monitor: a. urine output b. abdominal girth c. stool frequency d. level of consciousness (LOC)

Answer: Level of consciousness (LOC).

A physician orders spironolactone (Aldactone), 50 mg by mouth four times daily, for a client with fluid retention caused by cirrhosis. Which finding indicates that the drug is producing a therapeutic effect? Serum potassium level of 3.5 mEq/L Loss of 2.2 lb (1 kg) in 24 hours Blood pH of 7.25 Serum sodium level of 135 mEq/L

Answer: Loss of 2.2 lb (1 kg) in 24 hours

A physician orders spironolactone (Aldactone), 50 mg by mouth four times daily, for a client with fluid retention caused by cirrhosis. Which finding indicates that the drug is producing a therapeutic effect? a. Serum potassium level of 3.5 mEq/L b. Loss of 2.2 lb (1 kg) in 24 hours c. Blood pH of 7.25 d. Serum sodium level of 135 mEq/L

Answer: Loss of 2.2 lb (1 kg) in 24 hours

A client is admitted to the hospital with acute hemorrhage from esophageal varices. What medication should the nurse anticipate administering that will reduce pressure in the portal venous system and control esophageal bleeding? a. Vasopressin (Pitressin) b. Vitamin K c. Epinephrine d. Octreotide (Sandostatin)

Answer: Octreotide (Sandostatin)

After undergoing a liver biopsy, a client should be placed in which position? a. Prone position b. Supine position c. Right lateral decubitus position d. Semi-Fowler's position

Answer: Right lateral decubitus position

A client comes to the ED with severe abdominal pain, nausea, and vomiting. The physician plans to rule out acute pancreatitis. The nurse would expect the diagnosis to be confirmed by an elevated result on which laboratory test? a. Serum amylase b. Serum calcium c. Serum potassium d. Serum bilirubin

Answer: Serum amylase

The nurse should assess for an important early indicator of acute pancreatitis. What prolonged and elevated level would the nurse determine is an early indicator? a. Serum calcium b. Serum bilirubin c. Serum amylase d. Serum lipase

Answer: Serum lipase

A client who has been having recurrent attacks of severe abdominal pain over the past few months informs the physician about a 25-pound weight loss in the past year. The nurse attributes which factor as the most likely cause of this weight loss? a. Malabsorption b. Vomiting after heavy meals c. Skipping meals out of fear of painful attacks d. Ingesting a low-fat diet to prevent abdominal pain

Answer: Skipping meals out of fear of painful attacks

A client is admitted to the healthcare facility suspected of having acute pancreatitis and undergoes laboratory testing. Which of the following would the nurse expect to find? a. Increased serum calcium levels b. Elevated urine amylase levels c. Decreased liver enzyme levels d. Decreased white blood cell count

Answer: b. Elevated urine amylase levels

The nurse monitors a client for side effects associated with furosemide, which is newly prescribed for the treatment of heart failure. Complete the following sentence by choosing from the lists of options. Due to the client's high risk for developing ______ as a result of the prescribed medication, the nurse focuses on monitoring the client for _______.

Answer: hypokalemia, ventricular arrhythmia

Q: A patient has just been diagnosed with type 2 diabetes. The physician has prescribed an oral antidiabetic agent that will inhibit the production of glucose by the liver and thereby aid in the control of blood glucose. What type of oral antidiabetic agent did the physician prescribe for this patient? A) A sulfonylurea B) A biguanide C) A thiazolidinedione D) An alpha glucosidase inhibitor

B) A biguanide

Q: Which of the following patients should the nurse recognize as being at the highest risk for the development of osteomyelitis? A) A middle-age adult who takes ibuprofen daily for rheumatoid arthritis B) An elderly patient with an infected pressure ulcer in the sacral area C) A 17-year-old football player who had orthopedic surgery 6 weeks prior D) An infant diagnosed with jaundice

B) An elderly patient with an infected pressure ulcer in the sacral area

Q: A patient with a diagnosis of syndrome of inappropriate antidiuretic hormone secretion (SIADH) is being cared for in the critical care unit. The priority nursing diagnosis for a patient with this condition is what? A) Risk for peripheral neurovascular dysfunction B) Excess fluid volume C) Hypothermia D) Ineffective airway clearance

B) Excess fluid volume

Q: You are the surgical nurse caring for a 65-year-old female patient who is postoperative day 1 following a thyroidectomy. During your shift assessment, the patient complains of tingling in her lips and fingers. She tells you that she has an intermittent spasm in her wrist and hand and she exhibits increased muscle tone. What electrolyte imbalance should you first suspect? A) Hypophosphatemia B) Hypocalcemia C) Hypermagnesemia D) Hyperkalemia

B) Hypocalcemia

Q: The nurse is caring for a patient with Addison's disease who is scheduled for discharge. When teaching the patient about hormone replacement therapy, the nurse should address what topic? A) The possibility of precipitous weight gain B) The need for lifelong steroid replacement C) The need to match the daily steroid dose to immediate symptoms D) The importance of monitoring liver function

B) The need for lifelong steroid replacement

Q: In people with osteoporosis, compression of the vertebrae in the upper back can cause A) fluorosis. B) kyphosis. C) scoliosis. D) spina bifida.

B) kyphosis.

Q: The nurse is reviewing the laboratory values of a patient with bone pain for the last 4 days. Which abnormal finding would support osteomyelitis? Select all that apply. A) Elevated calcium B) Elevated white blood cell count (WBC) C) Elevated phosphorus D) Elevated erythrocyte sedimentation rate (ESR) E) Elevated C-reactive protein (CRP)

B, D, E

A nurse is checking laboratory values on a client who has crackles in the lower lobes, 2+ pitting edema, and dyspnea with minimal exertion. Which laboratory value does the nurse expect to be abnormal? Potassium B-type natriuretic peptide (BNP) C-reactive protein (CRP) Platelet count

B-type natriuretic peptide (BNP)

A neurologic assessment must be performed during t-PA administration: A. Every 5 minutes B. Every 15 minutes C. Every 30 minutes D. Every 60 minutes

B. Every 15 minutes

Q: A nurse is providing care for a client who has just been diagnosed with early-stage rheumatoid arthritis (RA). The nurse should anticipate the administration of which medication? A. Hydromorphone B. Methotrexate C. Allopurinol D. Prednisone

B. Methotrexate

FIXEDQ: The nurse caring for a patient with Cushing syndrome is describing the dexamethasone suppression test scheduled for tomorrow. What does the nurse explain that this test will involve? A) Administration of dexamethasone orally, followed by a plasma cortisol level every hour for 3 hours B) Administration of dexamethasone IV, followed by an x-ray of the adrenal glands C) Administration of dexamethasone orally at 11 PM, and a plasma cortisol level at 8 AM the next morning D) Administration of dexamethasone intravenously, followed by a plasma cortisol level 3 hours after the drug is administered

C) Administration of dexamethasone orally at 11 PM, and a plasma cortisol level at 8 AM the next morning

Q: A nurse is performing the health history and physical assessment of a patient who has a diagnosis of rheumatoid arthritis (RA). What assessment finding is most consistent with the clinical presentation of RA? A) Cool joints with decreased range of motion B) Signs of systemic infection C) Joint stiffness, especially in the morning D) Visible atrophy of the knee and shoulder joints

C) Joint stiffness, especially in the morning

Q: A nurse is educating a patient with gout about lifestyle modifications that can help control the signs and symptoms of the disease. What recommendation should the nurse make? A) Ensuring adequate rest B) Limiting exposure to sunlight C) Limiting intake of alcohol D) Smoking cessation

C) Limiting intake of alcohol

Q: Which instruction should a nurse give to a client with diabetes mellitus when teaching about "sick day rules"? A. "Don't take your insulin or oral antidiabetic agent if you don't eat." B. "It's okay for your blood glucose to go above 300 mg/dl while you're sick." C. "Test your blood glucose every 4 hours." D. "Follow your regular meal plan, even if you're nauseous."

C. "Test your blood glucose every 4 hours."

A patient arrives at the emergency department with symptoms of a stroke. Which diagnostic test should the nurse immediately prepare the patient for to further investigate the cause of the patient's symptoms? A- Cardiac marker levels B - Complete blood count (CBC) C - Computed tomography (CT) scan D - Transcranial doppler ultrasonography (TCD) Within: A- 5 mins B - 10 mins C - 15 mins D - 20 mins And give: A- IV alteplase B- IV adenosine C-PO alteplase D- IV lovenox Within: A-15 mins B- 30 mins C - 45 mins D- one hour

Computed tomography (CT) scan within 15 mins IV alteplase within 45 mins

A male client has been receiving a continuous infusion of weight-based heparin for more than 4 days. The client's PTT is at a level that requires an increase of heparin by 100 units per hour. The client has the laboratory findings shown above. The most important action of the nurse is to: Continue with the present infusion rate of heparin. Consult with the physician about discontinuing heparin. Begin treatment with the prescribed warfarin (Coumadin). Increase the heparin infusion by 100 units per hour.

Consult with the physician about discontinuing heparin.

A nurse assessing a client who underwent cardiac catheterization finds the client lying flat on the bed. The client's temperature is 99.8° F (37.7° C). The client's blood pressure is 104/68 mm Hg. The client's pulse rate is 76 beats/minute. The nurse detects weak pulses in the leg distal to the puncture site. Skin on the leg is cool to the touch. The puncture site is dry, but swollen. What is the most appropriate action for the nurse to take? Encourage the client to perform isometric leg exercise to improve circulation in the legs. Document findings and check the client again in 1 hour. Slow the IV fluid rate to prevent any more swelling at the puncture site. Contact the health care provider and report the findings.

Contact the health care provider and report the findings.

Q: A 49-year-old man with a history of poorly controlled type 1 diabetes has developed osteomyelitis adjacent to a chronic diabetic ulcer on his great toe. The patient has been informed that medical treatment for osteomyelitis requires a longer course of antibiotics than most other infections because: A) Osteomyelitis is usually caused by simultaneous infection with several microorganisms, which must be treated sequentially. B) Osteomyelitis requires treatment with topical antibiotics rather than IV antibiotics, necessitating a longer course of treatment. C) Osteomyelitis is usually the result of fungal infection rather than bacterial infection. D) Osteomyelitis involves the active infection of bone tissue, which is largely avascular.

D) Osteomyelitis involves the active infection of bone tissue, which is largely avascular.

Q: A diabetes nurse educator is presenting current recommendations for levels of caloric intake. What are the current recommendations that the nurse would describe? A. 10% of calories from carbohydrates, 50% from fat, and the remaining 40% from protein B. 10% to 20% of calories from carbohydrates, 20% to 30% from fat, and the remaining 50% to 60% from protein C. 20% to 30% of calories from carbohydrates, 50% to 60% from fat, and the remaining 10% to 20% from protein D. 50% to 60% of calories from carbohydrates, 20% to 30% from fat, and the remaining 10% to 20% from protein

D. 50% to 60% of calories from carbohydrates, 20% to 30% from fat, and the remaining 10% to 20% from protein

Q: A physician orders tests to determine if a client has systemic lupus erythematosus (SLE). Which test result helps to confirm an SLE diagnosis? A. Increased total serum complement levels B. Negative antinuclear antibody test C. Negative lupus erythematosus cell test D. An above-normal anti-deoxyribonucleic acid (DNA) test

D. An above-normal anti-deoxyribonucleic acid (DNA) test

Q: A client with diabetic ketoacidosis was admitted to the intensive care unit 4 hours ago and has these laboratory results: blood glucose level 450 mg/dl, serum potassium level 2.5 mEq/L, serum sodium level 140 mEq/L, and urine specific gravity 1.025. The client has two I.V. lines in place with normal saline solution infusing through both. Over the past 4 hours, his total urine output has been 50 ml. Which physician order should the nurse question? A. Infuse 500 ml of normal saline solution over 1 hour B. Hold insulin infusion for 30 minutes C. Add 40 mEq potassium chloride to an infusion of half normal saline solution and infuse at a rate of 10 mEq/hour D. Change the second I.V. solution to dextrose 5% in water

D. Change the second I.V. solution to dextrose 5% in water

Q: A nurse is reviewing the care of a client who has a long history of lower back pain that has not responded to conservative treatment measures. The nurse should anticipate the administration of what drug? A. Calcitonin B. Prednisone C. Aspirin D. Cyclobenzaprine

D. Cyclobenzaprine

Q: A client is suspected of having systemic lupus erythematosus. The nurse monitors the client, knowing that which of the following is one of the initial characteristic signs of systemic lupus erythematosus? A. Weight gain B. Subnormal temperature C. Elevated red blood cell count D. Rash on the face across the bridge of the nose and on the cheeks

D. Rash on the face across the bridge of the nose and on the cheeks

A client with rheumatic disease has developed a gastrointestinal (GI) bleed. The nurse caring for the client should further assess for medications that typically exacerbate this condition. Which medication applies? A. Corticosteroids B. Immunomodulators C. Antimalarials D. Salicylate therapy

D. Salicylate therapy

Q: A client with rheumatic disease has developed a gastrointestinal (GI) bleed. The nurse caring for the client should further assess for medications that typically exacerbate this condition. Which medication applies? A. Corticosteroids B. Immunomodulators C. Antimalarials D. Salicylate therapy

D. Salicylate therapy

11. A client diagnosed with hypernatremia needs fluid volume replacement. What intravenous solution would be the safest for the nurse to administer? a. 0.45% sodium chloride b. 0.9% sodium chloride c. 5% dextrose in water d. 5% dextrose in normal saline solution

a. 0.45% sodium chloride Rationale: A hypotonic solution (half-strength saline) is the solution of choice and considered safer than 5% dextrose in water because it allows a gradual reduction in the serum sodium level, thereby decreasing the risk of cerebral edema. An isotonic solution (0.9%) is not desirable as a supplement because it provides Na and CL.

6. The nurse should assess the patient for signs of lethargy, increasing intracranial pressure, and seizures when the serum sodium reaches what level? a. 115 mEq/L b. 130 mEq/L c. 145 mEq/L d. 160 mEq/L

a. 115 mEq/L Rationale: Features of hyponatremia associated with sodium loss and water gain include anorexia, muscle cramps, and a feeling of exhaustion. The severity of symptoms increases with the degree of hyponatremia and the speed with which it develops. When the serum sodium level decreases to less than 115 mEq/L (115 mmol/L), signs of increasing intracranial pressure, such as lethargy, confusion, muscle twitching, focal weakness, hemiparesis, papilledema, seizures, and death, may occur.

A nurse is caring for a client who experienced an MI. The client is ordered to received metoprolol. The nurse understands that this medication has which therapeutic effect? a. Decreases resting heart rate b. Decreases cholesterol level c. Increases cardiac output d. Decreases platelet aggregation

a. Decreases resting heart rate

38. A client has been admitted to the neurologic ICU with a diagnosis of a brain tumor. The client is scheduled to have a tumor resection/removal in the morning. Which of the following assessment parameters should the nurse include in the initial assessment? a. Gag reflex b. Deep tendon reflexes c. Abdominal girth d. Hearing acuity

a. Gag reflex

Q: Which reason best explains why diabetics are prone to infection? a. High glucose levels provide an environment conducive to bacterial growth. b. Atherosclerotic vascular changes decrease blood supply to tissues. c.Diabetics display abnormal phagocyte function. d. Diabetics display decreased leukocyte function.

a. High glucose levels provide an environment conducive to bacterial growth.

4. A client is experiencing edema in the tissue. What type of intravenous fluid would the nurse expect to be prescribed? a. hypertonic solution b. isotonic fluid c. no intravenous solution d. hypotonic solution

a. hypertonic solution Rationale: A hypertonic solution is used to pull water back into circulation, as it has more particles than the body's water. If hypertonics are given too rapidly or in large quantities, they may cause an extracellular volume excess and precipitate circulatory overload and dehydration. As a result, these solutions must be given cautiously and usually only when the serum osmolality has decreased to dangerously low levels. Hypertonic solutions exert an osmotic pressure greater than that of the extracellular fluid. The hospitalized client requires treatment for the tissue edema. An isotonic solution is the same concentration as the body's water and is used as an intravenous volume expander. A hypotonic solution has fewer particles than the body's water, thus shifting water from the vascular space to the tissue.

55. A nurse is giving a presentation on malpractice. Which statement indicates the nurse understands malpractice? a."The elements of duty, breach of duty, and patient injury must be present for a malpractice claim." b."Negligent nursing care and failure to follow standards must be present for a malpractice claim. c."Failure to report, defamation, and discrimination must be present for a malpractice claim." d."Error in judgment and invasion of privacy must be present for a malpractice claim."

a."The elements of duty, breach of duty, and patient injury must be present for a malpractice claim." Rationale: There are three elements that must be present for a malpractice claim: (1) You must have a duty—there must be a professional nurse-patient relationship. (2) You must have breached a duty that was foreseeable—you must have fallen below the standard of care. (3) Your breach of duty caused patient injury or damages. The other options do not indicate the nurse's understanding of malpractice.

43. The nurse practitioner prescribes the medication of choice for an MS patient who is experiencing disabling episodes of muscles spasms, especially at night. Which of the following is the drug most likely prescribed in this scenario? a. Dantrium b. Lioresal c. Zanaflex d. Valium

b. Lioresal Rationale: Baclofen (Lioresal), a gamma-aminobutyric acid (GABA) agonist, is the medication of choice for treating spasticity. It can be administered orally or by intrathecal injection.

Q: On assessment of a patient with early-stage hypothyroidism, the nurse practitioner assesses for a vague yet significant sign which is: a. Bradypnea b. Paresthesia c. Hypotension d. Hypothermia

b. Paresthesia

A nurse in the emergency department is caring for a client with acute heart failure. Which laboratory value is most important for the nurse to check before administering medications to treat heart failure? a. White blood cell (WBC) count b. Potassium c. Platelet count d. Calcium

b. Potassium

12. The nurse is reviewing the medical record of a client with glaucoma. Which of the following would alert the nurse to suspect that the client was at increased risk for this disorder? a. Hyperopia since age 20 years old b. Prolonged use of corticosteroids c. History of respiratory disease d. Age younger than 40 years old

b. Prolonged use of corticosteroids Rationale: Risk factors for glaucoma include thin cornea, African-American race, older age, diabetes mellitus, nearsightedness, and prolonged use of topical or systemic corticosteroids.

30. The clinical finding of pink frothy sputum may be an indication of which of the following? a. Bronchiectasis b. Pulmonary edema c. An infection d. A lung abscess

b. Pulmonary edema Rationale: Profuse, frothy pink material, often welling up into the throat, may indicate pulmonary edema. Foul-smelling sputum and bad breath may indicate a lung abscess, bronchiectasis, or an infection caused by fusospirochetal or other anaerobic organisms.

25. A nurse is caring for a client after a lung biopsy. Which assessment finding requires immediate intervention? a. O2 Sat of 96% on 3L of O2 b. RR of 42 breaths/min c. Client stating pain level of 7 out of 10 that decreases with pain medication d. Client dozing when left alone but wakening easily

b. RR of 42 breaths/min

A client reports chest pain that occurs when playing tennis but resolves when sitting down. The nurse knows these symptoms are common for which type of angina? a. Unstable b. Stable c. Intractable d. Variant

b. Stable

Q: A patient with polyarthralgia with joint swelling and pain is being evaluated for systemic lupus erythematosus (SLE). The nurse knows that the serum test result that is the most specific for SLE is the presence of a. rheumatoid factor. b. anti-Smith antibody (Anti-Sm). c. antinuclear antibody (ANA). d. lupus erythematosus (LE) cell prep.

b. anti-Smith antibody (Anti-Sm).

23. A Black client with asthma seeks emergency care for acute respiratory distress. Because of this client's dark skin, the nurse should assess for cyanosis by inspecting the: a. lips b. mucous membranes c. nail beds d. earlobes

b. mucous membranes

The nurse is presenting health education to a 48-year-old man who was just diagnosed with type 2 diabetes. The client has a BMI of 35 and leads a sedentary lifestyle. The nurse gives the client information on the risk factors for his diagnosis and begins talking with him about changing behaviors around diet and exercise. The nurse knows that further client teaching is necessary when the client tells you what? a. "I need to start slow on an exercise program approved by my doctor." b. "I know there's a chance I could have avoided this if I'd always eaten better and exercised more." c. "There is nothing that can be done anyway, because chronic diseases like diabetes cannot be prevented." d. "I want to have a plan in place before I start making a lot of changes to my lifestyle."

c. "There is nothing that can be done anyway, because chronic diseases like diabetes cannot be prevented."

42. A 33-year-old patient presents at the clinic with complaints of weakness, incoordination, dizziness, and loss of balance. The patient is hospitalized and diagnosed with MS. What sign or symptom, revealed during the initial assessment, is typical of MS? a. Diplopia, history of increased fatigue, and decreased or absent deep tendon reflexes b. Flexor spasm, clonus, and negative Babinski's reflex c. Blurred vision, intention tremor, and urinary hesitancy d. Hyperactive abdominal reflexes and history of unsteady gait and episodic paresthesia in both legs

c. Blurred vision, intention tremor, and urinary hesitancy Rationale: Optic neuritis, leading to blurred vision, is a common early sign of MS, as is intention tremor (tremor when performing an activity). Nerve damage can cause urinary hesitancy. In MS, deep tendon reflexes are increased or hyperactive. A positive Babinski's reflex is found in MS. Abdominal reflexes are absent with MS.

Q: A patient who experienced an open fracture of the humerus 2 weeks ago is having increased pain at the fracture site. To identify a possible causative agent of osteomyelitis at the site, what should the nurse expect testing to include? a. X-rays b. CT scan c. Bone biopsy d. WBC count and erythrocyte sedimentation rate (ESR)

c. Bone biopsy

49. The nurse is caring for a client who is hospitalized with an exacerbation of MS. To ensure the client's safety, what nursing action should be performed? a. Pad the client's bed rails. b. Maintain bed rest whenever possible. c. Ensure that suction apparatus is set up at the bedside. d. Provide several small meals each day.

c. Ensure that suction apparatus is set up at the bedside. Rationale: Because of the client's risk of aspiration, it is important to have a suction apparatus at hand. Bed rest should be generally be minimized, not maximized, and there is no need to pad the client's bed rails or to provide multiple small meals.

Q: Which patient assessment would indicate to the nurse that salicylate toxicity is occurring? a. Gastrointestinal (GI) bleeding b. Increased bleeding times c. Tinnitus d. Occasional nausea

c. Tinnitus

10. A nurse is assessing a client's reflexes. Which condition does the nurse need to confirm when tapping the facial nerve of a client who has dysphagia? a. hypervolemia b. hypercalcemia c. hypomagnesemia d. hypermagnesemia

c. hypomagnesemia Rationale: If there is a unilateral spasm of facial muscles when the nurse taps over the facial muscle, it is known as Chvostek's sign, which is a sign of hypocalcemia and hypomagnesemia. The additional symptom of dysphagia reinforces the possibility of hypomagnesemia rather than hypocalcemia. A positive Chvostek's sign does not apply to hypercalcemia, hypervolemia, or hypermagnesemia.

54. A medical-surgical nurse is aware of the scope of practice as defined in the jurisdiction where the nurse provides care. When exploring the legal basis for the scope of practice, the nurse should consult: a. codes of ethics. b. a code of nursing conduct. c. the nurse practice act in the nurse's jurisdiction. d. client preferences and norms within the profession.

c. the nurse practice act in the nurse's jurisdiction. Rationale: Nurses have a responsibility to comply with the nurse practice act of the jurisdiction in which they practice. A nurse's scope of practice is not determined by codes of ethics, codes of conduct, or client preferences.

41. A nurse is teaching a client who was recently diagnosed with myasthenia gravis. Which statement should the nurse include in her teaching? a. "You'll continue to experience progressive muscle weakness and sensory deficits." b. "You'll need to take edrophonium (Tensilon) to treat the disease." c. "The disease is a disorder of motor and sensory dysfunction." d. "This disease doesn't cause sensory impairment."

d. "This disease doesn't cause sensory impairment." Rationale: Myasthenia gravis affects motor function; therefore, the nurse should inform the client that sensory impairments won't occur. This disease is chronic; there's no cure. It can be managed with edrophonium in the diagnostic phase; however, this drug isn't used to treat the condition.

52. Which of the following best reflects the rationale for evidence-based practice? a. A method for determining reimbursement b. A process for accreditation c. A way to establish accountability d. A means to ensure quality care

d. A means to ensure quality care Rationale: Evidence-based practice (EBP) provides a means for ensuring quality care by identifying and evaluating current literature and research and incorporating the findings into client care. Quality assurance programs were developed and required for insurance reimbursement and for accreditation by the Joint Commission. The quality assurance programs sought to establish accountability to society on the part of the health professions for the quality, appropriateness, and cost of health services provided.

60. Your 90-year-old home care client designated her son to make decisions regarding her medical care when she is no longer able to do so. As she nears the end of her life, her son is consulted on an ever-increasing basis. What is the name of the legal instrument that activates her son's decision-making designation? a. Power of attorney b. Designated signer c. Living will d. Durable power of attorney

d. Durable power of attorney Rationale: A durable power of attorney (DPOA) for healthcare or healthcare proxy is the person the client designates to make medical decisions on the client's behalf when the client no longer can do so. It allows competent clients to identify exactly what life-sustaining measures they want to be implemented, avoided, or withdrawn and offers reassurance that others will carry out their wishes. This is a legal term used in a different context; a durable power of attorney (DPOA) for healthcare or healthcare proxy is the person the client designates to make medical decisions on the client's behalf when the client no longer can do so. A living will is a written or printed statement describing a person's wishes concerning medical care and life-sustaining treatments that are wanted or unwanted in the event that a person is unable to personally make those decisions. Although a living will describes a person's wishes, it does not designate decision-making power to another person in the same was as a DPOA. This is not a term used in healthcare

7. You are caring for a patient who is being treated on the oncology unit with a diagnosis of lung cancer with bone metastases. During your assessment, you note the patient complains of a new onset of weakness with abdominal pain. Further assessment suggests that the patient likely has a fluid volume deficit. You should recognize that this patient may be experiencing what electrolyte imbalance? a. Hypernatremia b. Hypomagnesemia c. Hypophosphatemia d. Hypercalcemia

d. Hypercalcemia Rationale: The most common causes of hypercalcemia are malignancies and hyperparathyroidism. Anorexia, nausea, vomiting, and constipation are common symptoms of hypercalcemia. Dehydration occurs with nausea, vomiting, anorexia, and calcium reabsorption at the proximal renal tubule. Abdominal and bone pain may also be present. Primary manifestations of hypernatremia are neurologic and would not include abdominal pain and dehydration. Tetany is the most characteristic manifestation of hypomagnesemia, and this scenario does not mention tetany. The patient's presentation is inconsistent with hypophosphatemia.

5. A client presents with muscle weakness, tremors, slow muscle movements, and vertigo. The following are the client's laboratory values: Sodium 134 mEq/L (134 mmol/L) Potassium 3.2 mEq/L (3.2 mmol/L) Chloride 111 mEq/L (111 mmol/L) Magnesium 1.1 mg/dL (0.45 mmol/L) Calcium 8.4 mg/dL (2.1 mmol/L) What fluid and electrolyte imbalance would the nurse relate to the client's findings? a. Hyponatremia b. Hypokalemia c. Hypocalcemia d. Hypomagnesemia

d. Hypomagnesemia Rationale: Magnesium, the second most abundant intracellular cation, plays a role in both carbohydrate and protein metabolism. The most common cause of this imbalance is loss in the gastrointestinal tract. Hypomagnesemia is a value less than 1.3 mg/dL (0.45 mmol/L). Signs and symptoms include muscle weakness, tremors, irregular movements, tetany, vertigo, focal seizures, and positive Chvostek's and Trousseau's signs.

14. The body responds to infection by increasing the production of white blood cells (WBCs). The nurse should evaluate the differential count for what type of WBCs, which are the first WBCs to respond to an inflammatory event? a. Basophils b. Eosinophils c. Monocytes d. Neutrophils

d. Neutrophils Rationale: Neutrophils, the most abundant type of white blood cell, are the first of the WBCs to respond to infection or inflammation. The normal value is 3,000 to 7,000/cmm (males) and 1,800 to 7,700/cmm (females).

Q: A nurse is assessing a client with a terminal illness and finds that the client has cachexia (wasting syndrome). The nurse interprets this as indicating which of the following? a. Extreme anorexia b. Severe asthenia c. Starvation d. Profound protein loss

d. Profound protein loss

44. The nurse is performing an initial assessment on a client who is admitted to rule out myasthenia gravis. Which of the following findings would the nurse expect to observe? a. Muscle weakness and hyporeflexia of the lower extremities b. Difficulty with urination c. Facial distortion and pain d. Ptosis and diplopia

d. Ptosis and diplopia Rationale: The initial manifestation of myasthenia gravis in two-thirds of clients involves the ocular muscles; diplopia and ptosis are common. Muscle weakness and hyporeflexia of the lower extremities are associated with Guillain-Barre syndrome. Facial distortion and pain are associated with Bell's palsy and tic douloureux.

24. On arrival at the intensive care unit, a critically ill female client suffers respiratory arrest and is placed on mechanical ventilation. The physician orders pulse oximetry to monitor the client's arterial oxygen saturation (SaO2) noninvasively. Which vital sign abnormality may alter pulse oximetry values? a. Tachypnea b. Fever c. Hypotension d. Tachycardia e. Hypotension

e. Hypotension Rationale: Hypotension, hypothermia, and vasoconstriction may alter pulse oximetry values by reducing arterial blood flow. Likewise, movement of the finger to which the oximeter is applied may interfere with interpretation of SaO2. All of these conditions limit the usefulness of pulse oximetry. Fever, tachypnea, and tachycardia don't affect pulse oximetry values directly.

A nurse is caring for a patient with rheumatoid arthritis who has been prescribed abatacept. Which drug-drug product can be safely administered to a patient who is receiving abatacept? A) Anakinra B) Etanercept C) Methotrexate D) Measles, mumps, and rubella (MMR) vaccine

ANS C: Methotrexate Professor R: RA may be treated with a nonbiologic or biologic disease-modifying antirheumatic drug (DMARD). Methotrexate is a nonbiologic DMARD that produces immune suppression and reduces inflammation. Because this drug can cause birth defects or miscarriages, a pregnancy test should be obtained before starting methotrexate.

Q: The nurse is teaching a client who is undergoing diagnostic tests for multiple myeloma. What clinical findings support the client's diagnosis of multiple myeloma? A.serum creatinine level 0.5 mg/dL B.serum calcium level of 7.5 mg/dL C.serum albumin level of 2.0 g/dL D.serum protein level 5.8 g/dL

ANS C: serum albumin level of 2.0 g/dL RATIONALE: Albumin is a protein found in the blood and low levels can be seen in myeloma. Normal albumin level is 3.4 to 5.4 g/dL. Serum creatinine level may be increased (above 1.2 mg/dL in men and 0.9 mg/dL in women). Serum calcium levels exceed 10.2 mg/dL in multiple myeloma because calcium is lost from the bone and reabsorbed in the serum. The serum protein level is increased in multiple myeloma.

A physical therapist has directed a physical therapist assistant to provide strengthening exercises for a patient with a painful arc of motion resulting from a chronically inflamed biceps brachii muscle. What is the MOST efficient type of exercise to utilize with this patient? A. Isometric exercises at the end of range of movement. B. Active concentric contractions through partial range of motion. C. Active eccentric contractions in the pain-free range. D. Isokinetic exercises through the full range of motion.

C. Active eccentric contractions in the pain-free range Professor R: Occupational and physical therapy provide education on pacing of activities, work simplification, range-of-motion exercises, and muscle-strengthening exercises. These therapies provide the client with the knowledge to actively participate in care.

Which information will the nurse include when teaching range-of-motion exercises to a patient with an exacerbation of rheumatoid arthritis? A- Affected joints should not be exercised when pain is present. B- Application of cold packs before exercise may decrease joint pain. C- Exercises should be performed passively by someone other than the patient. D- Walking may substitute for range-of-motion (ROM) exercises on some days.

Application of cold packs before exercise may decrease joint pain.

13. An older adult patient informs the nurse, "I don't see as well as I used to." What should the nurse explain to the patient about why vision becomes less efficient with age? (Select all the apply.) a. There is a decrease in pupil size b. There is slowing of accommodation c. There is an increase in lens opaqueness d. Most older patients develop glaucoma e. The optic nerve begins to degenerate

A,B,C Rationale: The lens enables focusing for near and distance vision through accommodation, the process by which the lens of the eye adjusts the focal length to focus a clear image on the retina. With aging and certain conditions (e.g., diabetes or trauma), the lens loses its transparency (becomes more opaque) and ability to focus (accommodation) due to the formation of a cataract.

Q: A client is seen in the office for reports of joint pain, swelling, and a low-grade fever. What blood studies does the nurse know are consistent with a positive diagnosis of rheumatoid arthritis (RA)? Select all that apply. A. Positive antinuclear antibody (ANA) B. Positive C-reactive protein (CRP) C. Aspartate aminotransferase (AST) and alanine transaminase (ALT) levels of 7 units/L D. Red blood cell (RBC) count of <4.0 million/mcL E. Red blood cell (RBC) count of >4.0 million/mcL

A,B,D

15. The nurse is reviewing lab work on a newly admitted client. Which diagnostic stud(ies) confirms the nursing problem statement of dehydration. Select all that apply. a. An elevated hematocrit level b. A low urine specific gravity c. Electrolyte imbalance d. Low protein level in the urine e. Absence of ketones in urine

A,C Rationale: Dehydration is a common primary or secondary diagnosis in health care. An elevated hematocrit level reflects low fluid level and a hemoconcentration. Electrolytes are in an imbalance as sodium and potassium levels are excreted together in client with dehydration. The urine specific gravity, due to concentrated particle level, is high. Protein is not a common sign of dehydration. Ketones are always present in the urine

16. The nurse is reviewing lab work on a newly admitted client. Which diagnostic stud(ies) confirms the nursing problem statement of dehydration. Select all that apply. a. An elevated hematocrit level b. A low urine specific gravity c. Electrolyte imbalance d. Low protein level in the urine e. Absence of ketones in urine

A,C Rationale: Remember, everyone has ketones in their urine, whether you have diabetes or not. (very few though) Ketones are chemicals made in your liver. They are high in patients with diabetes or with chronic vomiting, extreme exercise, low-carbohydrate diets, dehydration or eating disorders.

2. Which of the following are physiologic changes commonly seen in the older adult? Select all that apply: A) Decrease in blood flow to the kidneys thus a decrease in glomerular filtration. This leads to a decrease in medication excretion. B) Increase in peristalsis and faster gastric emptying thus enhancing absorption. C) Decrease in cardiac output leading to a decrease in drug delivery to tissues. D) Gastric secretions become more acidic thus absorbing medications more rapidly E) Decrease in hepatic enzyme function thus reducing and slowing metabolism of medications.

A,C,E Rationale: Normal age-related changes and diseases that alter blood flow, liver and renal function, or cardiac output may affect distribution and metabolism of medications in the older adult. Decreased renal blood flow and loss of functioning nephrons in the older adult decreases the rates of elimination and increases the duration of action. This increases the danger of drug accumulation and potential toxicity. Medications that can have prolonged renal function effects include digoxin, procainamide, and lithium.

58. Which of the following situations would be appropriate for ethic committee review? Select all that apply. a. Patient refusal of a lifesaving blood transfusion related to religious preference b. Restraining a patient, after all other viable options have been exhausted c. Request to administer fertilization injections to an infertile couple d. Institutional participation in gene chip technology directed at disease prevention e. Placing a 21-year-old cystic fibrosis patient on the double lung transplant list

A,D,E Rationale: Dilemmas that center on death and dying are prevalent in medical-surgical nursing practice. The client refusing blood because of a religious preference can be an ethical dilemma. A terminally ill client with a family member wanting or refusing life support is an ethical dilemma for the committee. Dilemmas that center on death and dying are prevalent in medical-surgical nursing practice. Allocation of organs to patients with an otherwise poor prognosis is an appropriate ethical issue for an ethics review committee. Advances in genetics and genetic testing provide controversial ethical dilemmas such as gene chip technology. Administering fertilization injections to infertile couples is an accepted medical intervention. The American Nurses Association (ANA) advocates for the placement of restraints "only when no other viable option is available." The Joint Commission and the Centers for Medicare and Medicaid Services (CMS) have published standards on the use of restraints.

Q: The nurse is describing some of the major characteristics of cancer to a patient who has recently received a diagnosis of malignant melanoma. When differentiating between benign and malignant cancer cells, the nurse should explain differences in which of the following aspects? Select all that apply. A) Rate of growth B) Ability to cause death C) Size of cells D) Cell contents E) Ability to spread

A.B.E R: Benign and malignant cells differ in many cellular growth characteristics, including the method and rate of growth, ability to metastasize or spread, general effects, destruction of tissue, and ability to cause death. Cells come in many sizes, both benign and malignant. Both benign and malignant cells can occur anywhere in the body.

A client with chronic renal failure complains of generalized bone pain and tenderness. Which assessment finding would alert the nurse to an increased potential for the development of spontaneous bone fractures? A. Hyperphosphatemia B. Hypercalcemia C. Hypophosphatemia D. Hypocalcemia

A: Hyperphosphatemia Professor R: Osteodystrophy is a condition in which the bone becomes demineralized due to hypocalcemia and hyperphosphatemia. In an effort to raise blood calcium levels, the parathyroid glands secrete more parathormone. Elevated creatinine, urea, nitrogen, and potassium levels are expected in chronic renal failure and do not contribute to bone fractures.

Q: A nurse is caring for a client with pain due to muscle spasm. Which nursing action is beneficial for the client? A)Providing heat compresses at the site B)Providing a massage to the affected area C)Encouraging the client to perform isometric exercises D)Encouraging the client to do active-passive range-of-motion (ROM) exercises

A: Providing heat compresses at the site

Q: The client is receiving a vesicant antineoplastic for treatment of cancer. Which assessment finding would require the nurse to take immediate action? a. Extravasation b. Stomatitis c. Nausea and vomiting d. Bone pain

ANS A: Extravasation R: The nurse needs to monitor IV administration of antineoplastics (especially vesicants) to prevent tissue necrosis to blood vessels, skin, muscles, and nerves. Stomatitis, nausea/vomiting, and bone pain can be symptoms of the disease process or treatment mode but does not require immediate action.

Q: You are the clinic nurse in an oncology clinic. Your patient arrives for a 2-month follow-up appointment following chemotherapy. You note that the patient's skin appears yellow. Which blood tests should be done to further explore this clinical sign? a) Liver function test b) CBC c) Platelet count d) Electrolytes

ANS A: LFT TEST PROFESSOR : Yellow skin is a sign of jaundice and the liver is a common organ affected by metastatic disease. An LFT should be done to determine if the liver is functioning.

As a client approaches death, respirations become noisy. This is the result of which type of physical event? Musculoskeletal change cardiac dysfunction central nervous system alterations gastrointestinal impairment

ANS A: Musculoskeletal change RATIONALE:As death approaches, a client's reflexes become hypoactive. The jaw and facial muscles also relax. As the tongue falls to the back of the throat, respirations become noisy.

Q: A client undergoes a biopsy of a suspicious lesion. The biopsy report classifies the lesion according to the TNM staging system as follows: TIS, N0, M0. What does this classification mean? \ A.) No evidence of primary tumor, no abnormal regional lymph nodes, and no evidence of distant metastasis B.) Carcinoma in situ, no abnormal regional lymph nodes, and no evidence of distant metastasis C.) Can't assess tumor or regional lymph nodes and no evidence of metastasis D.) Carcinoma in situ, no demonstrable metastasis of the regional lymph nodes, and ascending degrees of distant metastasis

ANS B: Carcinoma in situ, no abnormal regional lymph nodes, and no evidence of distant metastasis

Chemotherapeutic agents have different specific classifications. The following medications are antineoplastic antibiotics except: A.Doxorubicin (Adriamycin) B.Fluorouracil (Adrucil) C.Mitoxantrone (Novantrone) D.Bleomycin (Blenoxane)

ANS B: Fluorouracil (Adrucil) R: Fluorouracil (Adrucil) is an antimetabolite. PROFESSOR: Chemotherapeutic agents have different specific classifications. Antineoplastic antibiotics: Doxorubicin (Adriamycin), Mitoxantrone (Novantrone) and Bleomycin (Blenoxane).

A client is receiving the cell cycle-nonspecific alkylating agent thiotepa (Thioplex), 60 mg weekly for 4 weeks by bladder installation as part of chemotherapy regimen to treat bladder cancer. The client asks the nurse how the drug works. How does thiotepa exert its therapeutic effects? A.) It interferes with deoxyribonucleic acid (DNA) replication only. B.) It interferes with ribonucleic acid (RNA) transcription only. C.) It interferes with DNA replication and RNA transcription. D.) It destroys the cell membrane, causing lysis.

ANS C: It interferes with DNA replication and RNA transcription R: Thiotepa interferes with DNA replication and RNA transcription. It doesn't destroy the cell membrane.

A 35-year-old client is admitted to the hospital reporting severe headaches, vomiting, and testicular pain. The client's blood work shows reduced numbers of platelets, leukocytes, and erythrocytes, with a high proportion of immature cells. The nurse caring for this client suspects which diagnosis? A. Acute myeloid leukemia (AML) B. Chronic myeloid leukemia (CML) C. Myelodysplastic syndromes (MDS) D. Acute lymphocytic leukemia (ALL)

ANS D: Acute lymphocytic leukemia (ALL) RATIONALE: In acute lymphocytic leukemia (ALL), manifestations of leukemic cell infiltration into other organs are more common than with other forms of leukemia, and include pain from an enlarged liver or spleen, as well as bone pain. The central nervous system is frequently a site for leukemic cells; thus, clients may exhibit headache and vomiting because of meningeal involvement. Other extranodal sites include the testes and breasts. All types of leukemia, depending on severity and stage, can have the same blood work results. The difference is the client's signs and symptoms. A large number of clients when first diagnosed with any type of leukemia are asymptomatic or have nonspecific symptoms It is discovered on routine lab work.

Q: After cancer chemotherapy, a client experiences nausea and vomiting. The nurse should assign highest priority to which intervention? a) Withholding fluids for the first 4 to 6 hours after chemotherapy administration b) Serving small portions of bland food c) Encouraging rhythmic breathing exercises d) Administering metoclopramide and dexamethasone as ordered

ANS D: Administering metoclopramide and dexamethasone as ordered R: When your patient is experiencing Chemotherapy, you should assign highest priority to administering an antiemetic, such as metoclopramide, and an anti-inflammatory agent, such as dexamethasone, because it may reduce the severity of chemotherapy-induced nausea and vomiting. This intervention, in turn, helps prevent dehydration, a common complication of chemotherapy.

A female client is receiving methotrexate (Mexate), 12 g/m2 I.V., to treat osteogenic carcinoma. During methotrexate therapy, the nurse expects the client to receive which other drug to protect normal cells? a. probenecid (Benemid) b. cytarabine (ara-C, cytosine arabinoside [Cytosar-U]) c. thioguanine (6-thioguanine, 6-TG) d. leucovorin (citrovorum factor or folinic acid [Wellcovorin])

ANS D: Leucovorin (citrovorum factor or folinic acid [Wellcovorin]) R: Leucovorin is administered with methotrexate to protect normal cells, which methotrexate could destroy if given alone. Probenecid should be avoided in clients receiving methotrexate because it reduces renal elimination of methotrexate, increasing the risk of methotrexate toxicity.

Q: The nurse knows that interferon agents are used in association with chemotherapy to produce which effects in the client? A. Suppression of the bone marrow B. Enhance action of the chemotherapy C. Decrease the need for additional adjuvant therapies D. Shorten the period of neutropenia

ANS D: Shorten the period of neutropenia Rationale: Interferon agents are a type of biologic response modifiers (BRMs) used in conjunction with chemotherapy to reduce the risk of infection by shortening the period of neutropenia through bone marrow stimulation. The suppression of bone marrow creates the need for interferon use - not a result of the use. Although some BRMs can inhibit tumor growth, the primary use is for reducing neutropenia. Interferon use does not replace standard cancer treatments or decrease the need for those treatments.

The nursing educator is talking with a group of recent nursing graduates about common diagnoses on the unit. What diffuse connective tissue disease would the instructor tell the group is caused by an autoimmune reaction that results in phagocytosis, producing enzymes within the joint that break down collagen and cause edema? A. Rheumatoid arthritis (RA) B. Systemic lupus erythematosus (SLE) C. Osteoporosis D. Polymyositis

ANS: A. Rheumatoid arthritis (RA) Professor R: In RA, the autoimmune reaction results in phagocytosis, producing enzymes within the joint that break down collagen, cause edema and proliferation of the synovial membrane, and ultimately form pannus. Pannus destroys cartilage and bone.

Q: The nurse is providing care to a patient who has a decreased neutrophil count and elevated hepatic enzymes. Which data in the patient's health history supports this laboratory data indicating an increased risk for infection?

ANS: Cirrhosis of the liver (example of hepatic disease, leads to decreased neutrophil count, which leads to increased risk for infection.)

Q: A clinic nurse is caring for a patient with suspected gout. While explaining the pathophysiology of gout to the patient, the nurse should describe which of the following? A)Autoimmune processes in the joints B) Chronic metabolic acidosis C) Increased uric acid levels D) Unstable serum calcium levels

Ans C: Increased uric acid levels (Feedback: Gout is caused by hyperuricemia (increased serum uric acid). Gout is not categorized as an autoimmune disease and it does not result from metabolic acidosis or unstable serum calcium levels.)

A nurse is providing a class on osteoporosis at the local center for older adults. Which statement related to osteoporosis is most accurate? A. High levels of vitamin D can cause osteoporosis. B. A non modifiable risk factor for osteoporosis is a person's level of activity. C. Secondary osteoporosis occurs in women after menopause. D. The use of corticosteroids increases the risk of osteoporosis.

Ans D: The use of corticosteroids increases the risk of osteoporosis. Quizlet Rationale: Corticosteroid therapy is a secondary cause of osteoporosis when taken for long-term use. Adequate levels of vitamin D are needed for absorption of calcium. A person's level of physical activity is a modifiable factor that influences peak bone mass. Lack of activity increases the risk for the development of osteoporosis. Primary osteoporosis occurs in women after menopause.

17. The nurse is assisting a client with postural drainage. Which of the following demonstrates correct implementation of this technique? a. Perform this measure with the client once a day. b. Instruct the client to remain in each position of the postural drainage sequence for 10 to 15 minutes. c. Administer bronchodilators and mucolytic agents following the sequence. d. Use aerosol sprays to deodorize the client's environment after postural drainage.

B

34. A nursing student knows that there are three most common symptoms of asthma. Choose the three that apply. a. crackles b. cough c. wheezing d. dyspnea

B,C,D

A client with rheumatoid arthritis arrives at the clinic for a checkup. Which statement by the client refers to the most overt clinical manifestation of rheumatoid arthritis? A. "My legs feel weak." B. "My finger joints are oddly shaped." C. "I have pain in my hands." D. "I have trouble with my balance."

B: "My finger joints are oddly shaped." Professor R: Rheumatoid arthritis (RA), a chronic systemic autoimmune disorder of the connective tissue and joints characterized by remissions and exacerbations. The client's signs and symptoms of morning stiffness, spongy joints, and ulnar deviation are key findings in RA.

A client is receiving the cell cycle-nonspecific alkylating agent thiotepa (Thioplex), 60 mg weekly for 4 weeks by bladder installation as part of chemotherapy regimen to treat bladder cancer. The client asks the nurse how the drug works. How does thiotepa exert its therapeutic effects? A.) It interferes with deoxyribonucleic acid (DNA) replication only. B.) It interferes with ribonucleic acid (RNA) transcription only. C.) It interferes with DNA replication and RNA transcription. D.) It destroys the cell membrane, causing lysis.

C.) It interferes with DNA replication and RNA transcription.

19. A hospitalized client with terminal heart failure is nearing the end of life. The nurse observes which of the following breathing patterns?

Cheyne-Strokes Rationale: Cheyne-Stokes breathing is characterized by a regular cycle where the rate and depth of breathing increase, then decrease until apnea occurs. The duration of apnea varies but progresses in length. This breathing pattern is associated with heart failure, damage to the respiratory center in the brain, or both.

35. A nurse is caring for a patient with COPD. The patient's medication regimen has been recently changed and the nurse is assessing for therapeutic effect of a new bronchodilator. What assessment parameters suggest a consequent improvement in respiratory status? Select all that apply. a. Negative sputum culture b. Increased viscosity of lung secretions c. Increased respiratory rate d. Increased expiratory flow rate e. Relief of dyspnea

D,E

During a head-to-toe assessment of a patient with arthritis, you note bony outgrowths on the proximal interphalangeal joint. These outgrowths are known as __________ and occur in ______________.* A. Heberden's Node, osteoarthritis B. Bouchard's Node, rheumatoid arthritis C. Heberden's Node, rheumatoid arthritis D. Bouchard's Node, osteoarthritis

D. Bouchard's Node, osteoarthritis Professor R: Heberden and Bouchard nodes seen in osteoarthritis do not occur in RA. Also, osteoarthritis occurs in weight-bearing joints

18. A client is being treated in the ED for respiratory distress coupled with bacterial pneumonia. The client has no medical history. However, the client works in a coal mine and smokes 10 cigarettes a day. The nurse anticipates which order based on the client's immediate needs? a. Administration of antibiotics b. Completion of a 12-lead ECG c. Administration of corticosteroids and bronchodilators d. Client education: avoidance of irritants like smoke and pollutants

a. Administration of antibiotics Rationale: Antibiotics are administered to treat respiratory tract infections. Chronic bronchitis is inflammation of the bronchi caused by irritants or infection. Hence, smoking cessation and avoiding pollutants are necessary to slow the accelerated decline of the lung tissue. However, the immediate priority in this case is to cure the infection, pneumonia. Corticosteroids and bronchodilators are administered to asthmatic clients when they show symptoms of wheezing. An ECG is used to evaluate atrial arrhythmias.

45. A client with Guillain-Barre syndrome cannot swallow and has a paralytic ileus; the nurse is administering parenteral nutrition intravenously. The nurse is careful to assess which of the following related to intake of nutrients? a. Gag reflex and bowel sounds b. Condition of skin c. Respiratory status d. Urinary output and capillary refill

a. Gag reflex and bowel sounds Rationale: Paralytic ileus may result from insufficient parasympathetic activity. The nurse may administer parenteral nutrition and IV fluids. The nurse carefully assesses for the return of the gag reflex and bowel sounds before resuming oral nutrition. The other three choices are important assessment items, but not necessarily related to the intake of nutrients.

48. A client was undergoing conservative treatment for a herniated nucleus pulposus, at L5 - S1, which was diagnosed by magnetic resonance imaging. Because of increasing neurologic symptoms, the client undergoes lumbar laminectomy. The nurse should take which step during the immediate postoperative period? a. Logroll the client from side to side. b. Have the client sit up in a chair as much as possible. c. Elevate the head of the bed to 90 degrees. d. Discourage the client from doing any range-of-motion (ROM) exercises.

a. Logroll the client from side to side. Rationale: Logrolling the client maintains alignment of his hips and shoulders and eliminates twisting in his operative area. The nurse should encourage ROM exercises to maintain muscle strength. Because of pressure on the operative area, having the client sit up in a chair or with the head of the bed elevated should be allowed only for short durations.

29. An adult client is having his skin assessed. The client tells the nurse he has been a heavy smoker for the last 40 years. The client has clubbing of the fingernails. What does this finding tell the nurse? a. The client has chronic hypoxia b. The client has melanoma c. The client has COPD d. The client has asthma

a. The client has chronic hypoxia

A client has received several treatments of bleomycin. It is now important for the nurse to assess? a) Urine output b) Lung sounds c) Skin integrity d) Hand grasp

b) Lung sounds

Q: A student nurse is working with a client who is diagnosed with head trauma. The nurse has documented Cheyne-Stokes respirations. The student would expect to see which of the following? a) Periods of normal breathing followed by periods of apnea b) Regular breathing where the rate and depth increase, then decrease c) Period of cessation of breathing d) Irregular breathing at 14 to 18 breaths per minute

b) Regular breathing where the rate and depth increase, then decrease

9. What percentage of potassium excreted daily leaves the body by way of the kidneys? a. 40 b. 80 c. 20 d. 60

b. 80 Rationale: To maintain the potassium balance, the renal system must function, because 80% of the potassium excreted daily leaves the body by way of the kidneys. The other numerical values are incorrect.

53. A nurse is providing care for a patient who is postoperative day one following a bowel resection for the treatment of colorectal cancer. How can the nurse best exemplify the QSEN competency of quality improvement? a. By liaising with the members of the interdisciplinary care team b. By critically appraising the outcomes of care that is provided c. By integrating the patients preferences into the plan of care d. By documenting care in the electronic health record in a timely fashion

b. By critically appraising the outcomes of care that is provided Rationale: Evaluation of outcomes is central to the QSEN competency of quality improvements. Each of the other listed activities is a component of quality nursing care, but none clearly exemplifies quality improvement activities.

37. The clinic nurse caring for a patient with Parkinson's disease notes that the patient has been taking levodopa and carbidopa (Sinemet) for 7 years. For what common side effect of Sinemet would the nurse assess this patient? a. Pruritus b. Dyskinesia c. Lactose intolerance d. Diarrhea

b. Dyskinesia Rationale: Within 5 to 10 years of taking levodopa, most patients develop a response to the medication characterized by dyskinesia (abnormal involuntary movements). Another potential complication of long-term dopaminergic medication use is neuroleptic malignant syndrome characterized by severe rigidity, stupor, and hyperthermia. Side effects of long-term Sinemet therapy are not pruritus, lactose intolerance, or diarrhea.

28. The nurse is caring for a patient with chronic obstructive pulmonary disease (COPD). The patient has been receiving high-flow oxygen therapy for an extended time. What symptoms should the nurse anticipate if the patient were experiencing oxygen toxicity? a. Bradycardia and frontal headache b. Dyspnea and substernal pain c. Peripheral cyanosis and restlessness d. Hypotension and tachycardia

b. Dyspnea and substernal pain Rationale: Oxygen toxicity can occur when patients receive too high a concentration of oxygen for an extended period. Symptoms of oxygen toxicity include dyspnea, substernal pain, restlessness, fatigue, and progressive respiratory difficulty. Oxygen toxicity may occur when too high a concentration of oxygen is given for an extended period (generally longer than 24 hours). It is caused by overproduction of oxygen free radicals, which are by-products of cell metabolism. These free radicals then mediate a severe inflammatory response that can severely damage the alveolar capillary membrane leading to pulmonary edema and progressing to cell death. Signs and symptoms of oxygen toxicity include dyspnea, restlessness, and substernal discomfort, in addition to paresthesias, fatigue, malaise, progressive respiratory difficulty, refractory hypoxemia, alveolar atelectasis, and alveolar infiltrates evident on chest x-rays.

50. A home health nurse has completed a visit to a client and has immediately begun to document the visit. Accurate documentation that is correctly formatted is necessary for what reason? a. Guarantees that the nurse will not be legally liable for unexpected outcomes b. Ensures that the agency is correctly reimbursed for the visit c. Allows the client to gauge progress over time d. Facilitates safe delegation of care to unlicensed caregivers

b. Ensures that the agency is correctly reimbursed for the visit Rationale: The client's needs and the nursing care provided must be documented to ensure that the agency qualifies for payment for the visit. Documentation does not guarantee an absence of liability. Documentation is not normally provided to the client to gauge progress. Documentation is not primarily used to facilitate delegation to unlicensed caregivers.

1. A nursing instructor is preparing a class about age-related changes in the cardiovascular system that occur in the older adult. Which of the following would the instructor most likely include? a. Thinning of the heart valves b. Increased blood pressure c. Atrophy of the heart muscle d. Decreased arterial resistance

b. Increased blood pressure Rationale: Age-related changes in the cardiovascular system include thickening of the heart valves, increased blood pressure, hypertrophy of the heart muscle, and increased arterial resistance.

The nurse administers a dose of ramipril (Altace) 2.5 mg to a client at 9 am. While documenting administration of the medication, the nurse discovers that 1.25 mg, not 2.5 mg, was the prescribed dose. The nurse assesses the client, completes an incident report, and notifies the physician and nursing supervisor of the error. What statement does the nurse add to the client's record? a. Twice the amount of the prescribed ramipril was administered at 9 am. b. Ramipril (Altace] 2.5 mg was administered at 9 am. c. Client's blood pressure was 128/82 mm Hg after the administration of the incorrect dose of ramipril. d. An incident report was completed and filed.

b. Ramipril (Altace] 2.5 mg was administered at 9 am. Rationale: After an incident, the nurse would document a concise and objective description of what occurred, and any follow-up actions taken in the dient's record. The nurse would not document in the client's record that an incident report was completed. Nor would the nurse document that twice the prescribed dose was given or that an incorrect dose was given.

39. A client is diagnosed with amyotrophic lateral sclerosis (ALS) in the early stages. Which medication would the nurse most likely expect to be prescribed as treatment? a. Bromocriptine b. Riluzole c. Amantadine d. Benztropine mesylate

b. Riluzole

26. A client is chronically short of breath and yet has normal lung ventilation, clear lungs, and an arterial oxygen saturation SaO2 of 96% or better. The client most likely has: a. poor peripheral perfusion. b. a possible hematologic problem. c. left-sided heart failure. d. a psychosomatic disorder.

b. a possible hematologic problem. Rationale: SaO2 is the degree to which hemoglobin (Hb) is saturated with oxygen. It doesn't indicate the client's overall Hb adequacy. Thus, an individual with a subnormal Hb level could have normal SaO2 and still be shortness of breath, indicating a possible hematologic problem. Poor peripheral perfusion would cause subnormal SaO2. There isn't enough data to assume that the client's problem is psychosomatic. If the problem were left-sided heart failure, the client would exhibit pulmonary crackles.

8. A client with hypertension has been prescribed hydrochlorothiazide. What nursing action will best reduce the client's risk for electrolyte disturbances? a. Maintain a low sodium diet. b. Encourage fluid intake. c. Ensure the client has sufficient potassium intake. d. Encourage the use of over-the-counter calcium supplements.

c. Ensure the client has sufficient potassium intake. Rationale: Diuretics cause potassium loss, and it is important to maintain adequate intake during therapy. Hyponatremia is more of a risk than hypernatremia, so a low-sodium diet does not address the risk for electrolyte disturbances. There is no direct need for extra calcium intake and increased fluid intake does not reduce the client's risk for electrolyte disturbances. (Thiazide diuretics, such as hydrochlorothiazide, cause potassium loss, and it is important to maintain adequate intake during therapy. When taking thiazide diuretics, hyponatremia is more of a risk than hypernatremia.)

40. A patient is diagnosed with amyotrophic lateral sclerosis, also known as ALS or Lou Gehrig's disease. The nurse understands that the symptoms of the disease will begin in what way? a. Ascending paralysis b. Numbness and tingling in the lower extremities c. Weakness starting in the muscles supplied by the cranial nerves d. Jerky, uncontrolled movements in the extremities

c. Weakness starting in the muscles supplied by the cranial nerves Rationale: The chief symptoms are fatigue, progressive muscle weakness, cramps, fasciculations (twitching), and incoordination. In about 25% of patients, weakness starts in the muscles supplied by the cranial nerves, and difficulty in talking, swallowing, and ultimately breathing occurs.

Q: The nurse in the oncology clinic is caring for a 42-year-old female client receiving chemotherapy with fludarabine for acute myeloid leukemia who has developed petechiae, epistaxis, and ecchymosis. Complete the following sentence by choosing from the lists of options. ANS: The nurse anticipates that the client has developed_________ and that the laboratory results will reveal__________.

hemorrhage thrombocytopenia


Kaugnay na mga set ng pag-aaral

Leadership Theory and Development

View Set

AP Psych: Research Methods (Modules 4-8)

View Set

JMU merrell final hth 450 FORMULAS

View Set

CH. 14 Federal Court System Key Terms = Ms. Kirk's AP Gov't classes

View Set

N128 Week 3 - Adaptive Quizzing #3

View Set